Вы находитесь на странице: 1из 71

-cocci

Berry shaped bacteria.

-graph

Instrument to record.

-oid

Resembling

-ptosis

Prolapse

-scope

Instrument to visually examine.

-stomy

Surgical creation of a permanent


opening to the outside of the body.

-therapy

Treatment.

A 25-year-old client was admitted yesterday after a motor vehicle


collision. Neurodiagnostic studies showed a basal skull fracture in
the middle fossa. Assessment on admission revealed both halo and
Battle signs. Which new symptom indicates that the client is likely
to be experiencing a common life-threatening complication
associated with basal skull fracture?
A. Bilateral jugular vein distention.
B. Oral temperature of 102 degrees F.
C. Intermittent focal motor seizures.
D. Intractable pain in the cervical region.

B. Increased temp indicates


meningitis. (C & D) these symptoms
may be exhibited but are not life
threatening. (A) JVD is not a typical
complication of basal skull fractures.

39. Select all that apply. Which of the following are


clinical manifestations of tension pneumothorax?
A. Midline trachea
B. Severe hypertension
C. Progressive cyanosis
D. A loud bruit on affected side
E. Asymmetrical chest wall movement
F. Subcutaneous emphysema in the neck

C,E, F The indicators of tension


pneumothorax are asymmetrical
chest wall movement, severe
hypotension, subcutaneous
emphysema in the neck and upper
chest, and progressive cyanosis.

41. Select all that apply. During initial assessment, a


nurse should record which of the following
manifestations of respiratory distress?
A. Tachypnea
B. Nasal flaring
C. Thready pulse
D. Panting or grunting
E. Use of intercostal muscles
F. An inspiratory-to-expiratory ratio of 1:2

AD Manifestations of respiratory
distress include tachypnea, grunting
and panting on respiration, central
cyanosis, use of accessory muscles,
and flaring nares.

42. Select all that apply. Which of the following


nursing actions can help clear tracheobronchial
secretions in a patient with cystic fibrosis?
A. Postural drainage
B. Suppressing the cough
C. Ensuring adequate hydration
D. Administering mucolytic aerosols
E. Encouraging the patient to lie flat
F. Administering water-soluble vitamins
A 43-year-old homeless, malnourished female client with a history
of alcoholism is transferred to the ICU. She is placed on telemetry,
and the rhythm strip shown is obtained. The nurse palpates a heart
rate of 160 beats/min, and the client's blood pressure is 90/54.
Based on these finding, which IV medication should the nurse
administer?
A. Amiodarone (Cordarone)
B. Magnesium sulfate
C. Lidocaine (Xylocaine)
D. Procainamide (Pronestyl)

ACD Postural drainage, adequate


hydration, and administration of
mucolytic aerosols all encourage
coughing and the clearing of secretions.
A patient with cystic fibrosis will be more
comfortable sitting upright.
B. Because the client has chronic alcoholism, she is likely
to have hypomagnesium. (B) is the recommended drug
for torsades de pointes (AHA, 2005), which is a form of
polymorphic ventricular tachycardia (VT), usually
associated with a prolonged QT interval that occurs with
hypomagnesemia. (A and D) increase the QT interval,
which can cause the torsades to worsen. (C) is the
antiarrhythmic of choice in most cases of drug-induced
monomorphic VT, not torsades.

43. Select all that apply. Which of the following is


included in a comprehensive respiratory assessment?
A. Pulse oximetry
B. Chest auscultation
C. Apical radial pulse
D. Nail-bed assessment
E. Evaluation of respiratory effort
F. Rate and character of respirations

ABDEF The total assessment of the


respiratory system includes pulse
oximetry; auscultation; skin and nail-bed
assessment for the detection of cyanosis;
and rate, character, and degree of effort
of respirations. The apical radial pulse is
a cardiac assessment.

A 45-year-old man with asthma is brought to the


emergency department by automobile. He is short of
breath and appears frightened. During the initial nursing
assessment, which of the following clinical manifestations
might be present as an early symptom during an
exacerbation of asthma?
A. Anxiety
B. Cyanosis
C. Hypercapnia
D. Bradycardia

A. Anxiety An early symptom during an


asthma attack is anxiety because he is
acutely aware of the inability to get
sufficient air to breathe. He will be
hypoxic early on with decreased PaCO2
and increased pH as he is
hyperventilating.

A 55-year-old male client is admitted to the coronary


care unit having suffered an acute myocardial
infarction (MI). Within 24 hours of the occurrence,
the nurse can expect to find which systemic sign?

B. Tissue damage in the myocardium causes the release of


cardiac enzymes into the blood system. An elevated CMMB is a recognized indicator of an MI. It peaks 12 - 24
hours and returns to normal within 48 - 78 hours. (A)
would indicate pancreatitis or a gastric disorder. (D)
Although an elevated BUN might be related to an acute
MI it is usually associated with dehydration, high protein
intake or gastrointestinal bleeding and creatine levels
indicate renal damage. (C) Indicates effective
anticoagulation therapy.

A. Elevated serum amylase level


B. Elevated CM-MB level
C. Prolonged prothrombin time (PT)
D. Elevated serum BUN and creatinine

A 58-year-old client, who has no health problems, asks the nurse about taking
the pneumococcal vaccine (Pneumovax). Which statement give by the nurse
would offer the client accurate information about this vaccine?
A. "The vaccine is given annually before the flue season to those over 50 years
of age."
B. "The immunization is administered once to older adults or persons with a
history of chronic illness."
C. "The vaccine is for all ages and is given primarily to those person traveling
overseas to infected areas."
D. "The vaccine will prevent the occurrence of pneumococcal pneumonia for up
to 5 years."

B. It is usually recommended that persons


over 65 years of age and those with a history
of chronic illness should receive the vaccine
once in a lifetime. (A) the influenza vaccine is
given annually. (C) travel is not the main
rationale for the vaccine. (D) The vaccine is
usually given once in a lifetime.

A 71-year-old patient is admitted with acute respiratory distress


related to cor pulmonale. Which of the following nursing
interventions is most appropriate during admission of this patient?
A. Delay any physical assessment of the patient and review with
the family the patient's history of respiratory problems. B. Perform
a comprehensive health history with the patient to review prior
respiratory problems.
C. Perform a physical assessment of the respiratory system and ask
specific questions related to this episode of respiratory distress.
D. Complete a full physical examination to determine the effect of
the respiratory distress on other body functions.

C. Perform a physical assessment of the respiratory


system and ask specific questions related to this
episode of respiratory distress.Because the patient is
having respiratory difficulty, the nurse should ask
specific questions about this episode and perform a
physical assessment of this system. Further history
taking and physical examination of other body
systems can proceed once the patient's acute
respiratory distress is being managed.

A 75-year-old obese patient who is snoring


loudly and having periods of apnea several
times each night is most likely experiencing
A. narcolepsy.
B. sleep apnea.
C. sleep deprivation.
D. paroxysmal nocturnal dyspnea.

B. sleep apnea. Sleep apnea is most common in obese


patients. Typical symptoms include snoring and
periods of apnea. Narcolepsy is when a patient falls
asleep unexpectedly. Sleep deprivation could result
from sleep apnea. Paroxysmal nocturnal dyspnea
occurs when a patient has shortness of breath during
the night.

absorption atelectasis

Absorption of vitamin B12 may be decreased in older


adults because of decreased
A. intestinal motility.
B. production of bile by the liver.
C. production of intrinsic factor by the stomach.
D. synthesis of cobalamin (vitamin B12) by intestinal
bacteria.

alveolar collapse that occurs when high


concentrations of oxygen are given and
oxygen replaces nitrogen in the alveoli; if
airway obstruction occurs, the oxygen is
absorbed into the bloodstream and the
alveoli collapse.
C. production of intrinsic factor by the stomach. Older persons are at risk for
deficiency of cobalamin (pernicious anemia) because of a naturally occurring
reduction of the intrinsic factor by the stomach mucosa. Absorption of
cobalamin relies on intrinsic factor. Both must be present for absorption.
Megaloblastic anemia is related to folate dysfunction. Intestinal motility
(peristalsis) is the motion that moves food down the GI tract. The rhythmic
contractions of muscles cause wave-like motions. Lack of peristalsis is called
"paralytic ileus." Bile is produced in the liver, is stored and concentrated in the
gallbladder, and is released into the duodenum when fat is eaten. Bile
emulsifies fats and prepares them for enzyme digestion in order for the
nutrient to be absorbed into lymph and eventually into blood vessels to the
liver. Vitamin K (the blood-clotting vitamin) is synthesized by intestinal
bacteria.

ACE inhibitor

cough os a common side effect


hypertension
do not stop abruptly (rebound
hypertension may occur)

Acromegaly

Enlargement of extremities after


puberty due to pituitary gland
problem.

acute bronchitis

an inflammation of the lower


respiratory tract that is usually due to
infection.

Adipose

Pertaining to fat.

adventitious sounds

extra breath sounds that are not


normally heard, such as crackles,
rhonchi, wheezes, and pleural friction
rubs.

After a posterior nasal pack is inserted by a physician, the


patient is very anxious and states, "I don't feel like I'm
breathing right." The immediate intervention the nurse
should initiate is to
A. monitor ABGs.
B. reassure the patient that this is normal discomfort.
C. cut the pack strings and pull the packing out with a
hemostat.
D. direct a flashlight into the patient's mouth and inspect
the oral cavity.

D. direct a flashlight into the patient's mouth and


inspect the oral cavity. The nurse should inspect the
oral cavity for the presence of blood, soft palate
necrosis, and proper placement of the posterior plug.
If the posterior plug is visible, the physician should
be notified for readjustment of the packing.
Reassurance, cutting the strings, and ABGs are not
top priority interventions. The nurse needs further
data before intervening.

After admitting a patient to the medical unit with a


diagnosis of pneumonia, the nurse will verify that
which of the following physician orders have been
completed before administering a dose of cefotetan
(Cefotan) to the patient?
A. Serum laboratory studies ordered for AM
B. Pulmonary function evaluation
C. Orthostatic blood pressures
D. Sputum culture and sensitivity

D. Sputum culture and sensitivityThe nurse should ensure that the


sputum for culture and sensitivity was sent to the laboratory before
administering the cefotetan. It is important that the organisms are
correctly identified (by the culture) before their numbers are affected
by the antibiotic; the test will also determine whether the proper
antibiotic has been ordered (sensitivity testing). Although
antibiotic administration should not be unduly delayed while
waiting for the patient to expectorate sputum, all of the other
options will not be affected by the administration of antibiotics.

Aldosteronism

allergic rhinitis

alpha 1-antitrypsin

lab =decreased serum level of


potassium
hypokalemia
hypertension

the reaction of the nasal mucosa to a


specific allergen.

a serum protein produced by the liver


normally found in the lungs that
inhibits proteolytic enzymes of white
cells from lysing lung tissue; genetic
deficiency of this protein can cause
emphysema.

Amniocentesis

Surgical puncture to remove fluid


from the sac around the embryo.

angina pectoris

when walking=1.assist to seated


position; 2. sublingual nitroglycerin;
3.oxygen 4. wheelchair to room

Angioplasty

Surgical repair of blood vessel.

Anticoagulant therapy is used in the


treatment of thromboembolic disease
because anticoagulants can
A. dissolve the thrombi.
B. decrease blood viscosity.
C. prevent absorption of vitamin K.
D. inhibit the synthesis of clotting factors.

apnea

D. inhibit the synthesis of clotting factors.


Anticoagulant therapy is based on the premise that
the initiation or extension of thrombi can be
prevented by inhibiting the synthesis of clotting
factors or by accelerating their inactivation. The
anticoagulants heparin and warfarin do not induce
thrombolysis but effectively prevent clot extension.

an absence of spontaneous
respirations.

The arterial blood gas (ABG) readings that indicate


compensated respiratory acidosis are a PaCO2 of
A. 30 mm Hg and bicarbonate level of 24 mEq/L.
B. 30 mm Hg and bicarbonate level of 30 mEq/L.
C. 50 mm Hg and bicarbonate level of 20 mEq/L.
D. 50 mm Hg and bicarbonate level of 30 mEq/L.

Arteriole

asthma

D. 50 mm Hg and bicarbonate level


of 30 mEq/L. If compensation is
present, carbon dioxide and
bicarbonate are abnormal (or nearly
so) in opposite directions (e.g., one is
acidotic and the other alkalotic).

Small artery.

a chronic inflammatory lung disease that


results in airflow obstruction;
characterized by recurring episodes of
paroxysmal dyspnea, wheezing on
expiration and/or inspiration caused by
constriction of the bronchi, coughing,
and viscous mucoid bronchial secretions.

Before beginning a transfusion of RBCs, which of the following actions by the nurse would be of
highest priority to avoid an error during this procedure?
A. Check the identifying information on the unit of blood against the patient's ID bracelet.
B. Select new primary IV tubing primed with lactated Ringer's solution to use for the
transfusion.
C. Add the blood transfusion as a secondary line to the existing IV and used the IV controller to
maintain correct flow. D. Remain with the patient for 60 minutes after beginning the transfusion
to watch for signs of a transfusion reaction. The patient's identifying information (name, date of
birth, medical record number) on the identification bracelet should exactly match the
information on the blood bank tag that has been placed on the unit of blood. If any information
does not match, the transfusions should not be hung because of possible error and risk to the
patient.

A. The patient's identifying information (name,


date of birth, medical record number) on the
identification bracelet should exactly match the
information on the blood bank tag that has been
placed on the unit of blood. If any information
does not match, the transfusions should not be
hung because of possible error and risk to the
patient.

Before discharge, the nurse discusses activity levels with a


61-year-old patient with COPD and pneumonia. Which of
the following exercise goals is most appropriate once the
patient is fully recovered from this episode of illness?
A. Slightly increase activity over the current level.
B. Walk for 20 minutes a day, keeping the pulse rate less
than 130 beats per minute. C. Limit exercise to activities
of daily living to conserve energy.
D. Swim for 10 min/day, gradually increasing to 30
min/day.

B. Walk for 20 minutes a day, keeping the pulse rate


less than 130 beats per minute. The patient will
benefit from mild aerobic exercise that does not
stress the cardiorespiratory system. The patient
should be encouraged to walk for 20 min/day,
keeping the pulse rate less than 75% to 80% of
maximum heart rate (220 minus patient's age).

Before starting a transfusion of packed red blood


cells for an anemic patient, the nurse would arrange
for a peer to monitor his or her other assigned
patients for how many minutes when the nurse
begins the transfusion?
A. 60
B. 5
C. 30
D. 15

Bell palsy (7th crainal nerve)

D. 15 As part of standard procedure, the


nurse remains with the patient for the
first 15 minutes after hanging a blood
transfusion. Patients who are likely to
have a transfusion reaction will more
often exhibit signs within the first 15
minutes that the blood is infusing.

unilateral facial weakness and


paralysis

The blood bank notifies the nurse that the two units of blood
ordered for an anemic patient are ready for pick up. The nurse
should take which of the following actions to prevent an adverse
effect during this procedure?
A. Immediately pick up both units of blood from the blood bank.
B. Regulate the flow rate so that each unit takes at least 4 hours to
transfuse.
C. Set up the Y-tubing of the blood set with dextrose in water as the
flush solution.
D. Infuse the blood slowly for the first 15 minutes of the
transfusion.

D. Infuse the blood slowly for the first 15 minutes of


the transfusion. Because a transfusion reaction is
more likely to occur at the beginning of a transfusion,
the nurse should initially infuse the blood at a rate no
faster than 2 ml/min and remain with the patient for
the first 15 minutes after hanging a unit of blood.

blunt trama to back of head

LOC assessment most important

cancer

reduce fats
increase fruits, vegetables and fiber
ie bran flakes, skim milk, orange
slices

Catabolism

The process by which food is burned


to realease energy.

Cell Membrane

Allows materials to pass into and out


of the cell.

A central venous catheter has been inserted via a jugular vein and a
radiography has confirmed placement of the catheter. A
prescription has been received for stat medication but IV fluids
have not yet been started. What action should the nurse take prior
to administering the prescribed medication?
A. Assess for signs of jugular vein distention.
B. Obtain the needed intravenous solution.
C. Administer a bolus of normal saline solution.
D. Flush the line with heparinized saline.

centrilobular emphysema

chemoreceptor

C. A medication can be administered central line


without IV fluids, flush with normal saline to remove
heparin that may counteract with the medication. (B)
is used following the medication and a second saline
bolus. (A) will not impact the the med administration
and is not a priority. (B) Administration of the stat
medication is more of a priority than (B).

type of emphysema often associated with


chronic bronchitis in which respiratory
bronchioles enlarge, the walls are destroyed,
and the bronchioles become confluent;
characterized by enlargement of air spaces in
the proximal part of the acinus, primarily at
the level of the respiratory bronchioles.

a sensory nerve cell that responds to


a change in the chemical composition
(PaCO2 and pH) of the fluid around
it.

chest percussion

chest physiotherapy

rhythmic percussion of a patient's


chest with cupped hands to loosen
retained respiratory secretions.

a series of maneuvers including


percussion, vibration, and postural
drainage designed to promote
clearance of excessive respiratory
secretions.

chest tube

decreased drainage =assess for kinks


or dependant loops -do not clamp off

Chronic

Continuing over a long period of


time.

chronic bronchitis

obstructive pulmonary disease


characterized by excessive production of
mucus and chronic inflammatory
changes in the bronchi, resulting in a
cough with expectoration for at least 3
months of the year for more than 2
consecutive years.

The chronic inflammation of the bronchi


characteristic of chronic obstructive pulmonary
disease (COPD) results in
A. collapse of small bronchioles on expiration.
B. permanent, abnormal dilation of the bronchi.
C. hyperplasia of mucus-secreting cells and bronchial
edema.
D. destruction of the elastic and muscular structures
of the bronchial wall.

chronic pancreatitis

chylothorax

cirrhosis

CKD chronic kidney disease

C. hyperplasia of mucus-secreting cells and bronchial


edema. Chronic bronchitis is characterized by chronic
inflammation of the bronchial lining, with edema and
increased mucus production. Collapse of small
bronchioles on expiration is common in emphysema,
and abnormal dilation of the bronchi because of
destruction of the elastic and muscular structures is
characteristic of bronchiectasis.

pulmonary disease state characterized by the


presence of airflow obstruction caused by
chronic bronchitis or emphysema; clinical
use of the term indicates the presence of
chronic bronchitis and/or emphysema;
includes asthma, chronic bronchiectasis,
chronic bronchitis, and emphysema.

a condition marked by lymphatic


fluid in the pleural space caused by a
leak in the thoracic duct.

Vitamin K1 (AquaMephyton)
high calorie, low sodium diet
sodium restriction w/ edema
fluids restricted to decrease ascites
late stage = ascites

prior to hemodialysis lab=


hypocalcemia due to
hyperphosphatemia, hyperkalemic &
hypernatremic

A client diagnosed with angina pectoris complains of


chest pain while ambulating in the hallway. Which
action should the nurse implement first?
A. Support the client to a sitting position.
B. Ask the client to walk slowly back to the room.
C. Administer a sublingual nitroglycerin tablet.
D. Provide oxygen via nasal cannula.

A client diagnosed with chronic kidney disease (CDK) 2


years ago is regularly treated at a community
hemodialysis facility. In assessing the client before his
scheduled dialysis treatment, which electrolyte imbalance
should the nurse anticipate?
A. Hypophosphatemia
B. Hypocalcemia
C. Hyponatremia
D. Hypokalemia

A client is placed on a mechanical ventilator following a cerebral


hemorrhage, and vecuronium bromide (Norcuron) 0.04 mg/kg
every 12 hours IV is prescribed. What is the priority nursing
diagnosis for this client?
A. Impaired communication related to paralysis of skeletal
muscles.
B. Hight risk or infection related to increased ICP.
C. Potential for injury related to impaired lung expansion.
D. Social isolation related to inability to communicate.

A client who is receiving an ACE inhibitor for hypertension calls the


clinic and reports the recent onset of a cough to the nurse. What action
should the nurse implement?
A. Advise the client to come to the clinic immediately for further
assessment.
B. Instruct the client to discontinue use of the drug, and make an
appointment at the clinic.
C. Suggest that the client lear to accept the cough as a side effect to a
necessary prescription.
D. Encourage the client to keep taking the drug until seen by the HCP.

A client with alcohol-related liver disease is admitted


to the unit. Which prescription should the nurse
question as possibly inappropriate for the client?
A. Vitamin K1 (AquaMEPHYTON) 5 mg IM daily
B. High-calorie, low-sodium diet
C. Fluid restriction to 1500 ml/day
D. Pentobarbital (Nembutal sodium) 50 mg at
bedtime for rest

A. Assist in safely repositioning and


then administer (C & D). Then the
client can be escorted back to the
room via wheelchair or stretcher (B).

B. Hypocalcemia develops in CKD


due to chronic hyperphosphatemia
not (A). (C & D) incorrect you would
find hypernatremia and
hyperkalemia
A. To increase the client's tolerance of the
endotracheal intubation and/or mechanical
ventilation, a skeletal-muscle relaxant such as
vecuronium is usually prescribed. (A) is a serious
outcome because the client cannot communicate
his/her needs. (D) is not as much of a priority. (B)
infection is not related to ICP. (C) is incorrect
because the ventilator will ensure that the lungs are
expanded.

D. Cough is a common s/e of ACE inhibitors and is


not an indication to discontinue the medication. (A)
immediate evaluation is not needed. (B) an
antihypertensive should not be stopped abruptly. (C)
is demeaning since the cough may be disruptive to
the client and other medications may produce results
without the s/e.

D. Sedatives such as Nembutal are contraindicated


for clients with liver damage and can have dangerous
consequences. (A) is often prescribed since normal
clotting mechanism is damaged. (B) is needed to
restore energy. (C) Fluids are restricted to decrease
ascites which often accompanies cirrhosis,
particularly in later stages of the disease.

A client with chronic asthma is admitted to postanesthesia complaining


of pain at level 8 of 10, with a BP of 124/78, pulse of 88 beats/min, and
respirations of 20 breaths/min. The postanesthesia recovery
prescription is, "Morphine 2 to 4 mg IV push while in recovery for pain
level over 5." What intervention should the nurse implement?
A. Give the medication as prescribed to decrease the client's pain.
B. Call the anesthesia provider for a different medication for pain.
C. Use nonpharmacologic techniques before giving the medication.
D. Reassess pain level in 30 Minutes and medicate if it remains
elevated.

A client with cirrhosis states that his disease was


cause by a blood transfusion. What information
should the nurse obtain first to provide effective
client teaching?
A. The year the blood transfusion was received
B. The amount of alcohol the client drinks
C. How long the client has had cirrhosis
D. The client's normal coping mechanisms

A client with hypertension has been receiving ramipril (Altace) 5


mg PO daily for 2 weeks and is scheduled to receive a dose at 0900.
At 0830 the client's blood pressure is 120/70. Which action should
the nurse take?
A. Administer the dose as prescribed.
B. Hold the dose and contact the healthcare provider.
C. Hold the dose and recheck the blood pressure in 1 hour.
D. Check the healthcare provider's prescription to clarify the dose.

Coccyx

community-acquired pneumonia

B. Call for a different medication because


morphine and meperidine (Demerol)
have histamine-releasing narcotics and
should be avoided when a client has
asthma. (A) puts the client at risk for
asthma attack. (C & D) disregard the
clients prescription and pain relief.
A. The nurse should first verify the clients
explanation (A) since it may be accurate
due to prior to 1990 blood was not
screened for Hep C and hep C can cause
cirrhosis. Not all cirrhosis is caused is
caused by alcoholism (B) (C & D) provide
useful but less relevant information.

A. The BP is WNL and indicates that


the medication is working. (B & C)
would be indicated if the BP was low
(systole below 100). (D) is not
required because the dose is within
manufacture's recommendations.

Tailbone.

a lower respiratory tract infection of


the lung parenchyma with onset in
the community or during the first 2
days of hospitalization.

compliance

a measure of the ease of expansion of


the lungs and thorax.

COPD

contributing factor=smoking

cor pulmonale

CPR

crackle

hypertrophy of the right side of the


heart, with or without heart failure,
resulting from pulmonary
hypertension.

just above the xiphoid process on the


lower third of the sternum

short, low-pitched sounds consisting of


discontinuous bubbling caused by air
passing through airway intermittently
occluded by mucus, unstable bronchial wall,
or fold of mucosa; evident on inspiration
and, at times, expiration; similar sound to
blowing through a straw under water.

Craniotomy

Cushing Syndrome

cystic fibrosis

Cystocele

Debilitating anginal pain can be decreased in some clients by the


administration of beta-blocking agents such as nadolol (Corgard). Which
client requires the nurse to use extreme caution when administering
Corgard?
A. A 56-year-old air traffic controller who had bypass surgery 2 years
ago.
B. A 47-year-old kindergarten teacher diagnosed with asthma 40 years
ago
C. A 52-year-old unemployed stock broker who refuses treatment for
alcoholism
D. A 60-year-old retired librarian who takes a diuretic daily for
hypertension.

Incision of the skull.

results from hypersecreation


glucocorticoids in the adrenal cortex
often develope diabetes mellitus monitor serum glucose levels
generialized edema
low calorie, low carbohydrate, low
sodium diet

an autosomal recessive, multisystem


disease characterized by altered
function of the exocrine glands
involving primarily the lungs,
pancreas, and sweat glands.

Hernia of the urinary bladder.

B. asthma must be carefully monitored


because beta blockers because it can
induce cardiogenic shock and reduce
bronchodilation efforts. (A & D) this
medication is indicated and (C) it is not
contraindicated.

deviated septum

a deflection of the normally straight


nasal septum.

Diaphragm

Muscular wall separating the


abdominal and thoracic cavities.

diaphragmatic breathing

breathing with the use of the


diaphragm to achieve maximum
inhalation and slow respiratory rate.

digitalis (Lanoxin)
digoxin

Disc

cardiac glycoside
can build up toxic levels s/s anexoria,
nausea, vomiting, diarrhea,
headache, fatigue

A piece of cartilage between


backbones.

diverticulitits

hard ridgid abdomen & elevated WBC =


peritonitis = medical emergency should
be reported to PCP immediately
s/s left lower quadrant pain; elevated
temperature; refusing to eat; nausea

DNA

Genes are composed of?

dry powder inhaler

dry powdered drug delivered by


inhalation.

During admission of a patient diagnosed with nonsmall cell carcinoma of the lung, the nurse questions
the patient related to a history of which of the
following risk factors for this type of cancer? (Select
all that apply.)
A. Asbestos exposure
B. Cigarette smoking
C. Exposure to uranium
D. Chronic interstitial fibrosis

A,B,C Non-small carcinoma is associated


with cigarette smoking and exposure to
environmental carcinogens, including
asbestos and uranium. Chronic
interstitial fibrosis is associated with the
development of adenocarcinoma of the
lung.

During assessment of a 45-year-old patient with


asthma, the nurse notes wheezing and dyspnea. The
nurse interprets that these symptoms are related to
which of the following pathophysiologic changes? A.
Laryngospasm B. Overdistention of the alveoli C.
Narrowing of the airway D. Pulmonary edema

C. Narrowing of the airwayNarrowing


of the airway leads to reduced
airflow, making it difficult for the
patient to breathe and producing the
characteristic wheezing.

During assessment of a client in the intensive care unit,


the nurse notes that the client's breath sounds are clear
upon auscultation, but jugular vein distention and
muffled heart sounds are present. Which intervention
should the nurse implement?
A. Prepare the client for a pericardial tap.
B. Administer intravenous furosemide (Lasix).
C. Assist the client to cough and deep breathe.
D. Instruct the client to restrict oral fluid intake.

A. The client is exhibiting symptoms of


cardiac tamponade that results in
reduced cardiac output. Treatment is
pericardial tap. (B) is not a treatment.
(C) is not priority. (D) Fluids are
frequently increased but this is not as
priority as (A).

During care of a patient with multiple myeloma, an


important nursing intervention is
A. limiting activity to prevent pathologic fractures.
B. assessing for changes in size and characteristics of
lymph nodes.
C. maintaining a fluid intake of 3 to 4 L/day to dilute
calcium load.
D. administering narcotic analgesics continuously to
control bone pain.

C. maintaining a fluid intake of 3 to 4


L/day to dilute calcium load. Adequate
hydration must be maintained to
minimize problems from hypercalcemia.
The goal of a urinary output of 1.5 to 2
L/day requires an intake of 3 to 4 L/day.

During discharge teaching for a 65-year-old patient


with emphysema and pneumonia, which of the
following vaccines should the nurse recommend the
patient receive?
A. S. aureus
B. H. influenzae
C. Pneumococcal
D. Bacille Calmette-Gurin (BCG)

C. Pneumococcal The pneumococcal


vaccine is important for patients with
a history of heart or lung disease,
recovering from a severe illness, age
65 or over, or living in a long-term
care facility.

During report, the nurse learns that a client with tumor lysis
syndrome is receiving an IV infusion containing insulin. Which
assessment should the nurse complete first?
A. Review the client's history for diabetes mellitus.
B. Observe the extremity distal to the IV site.
C. Monitor the client's serum potassium and blood glucose.
D. Evaluate the client's oxygen saturation and breath sounds.

During the change of shift report, the


charge nurse reviews the infusions
being received by the clients on the
oncology unit. The client receiving
which infusion should be seen first?

C. The client with tumor lysis


syndrome may experience
hyperkalemia, therefor it is important
to monitor serum potassium and
blood glucose levels. (A, B, D) are not
as priority.

C. Has the highest risk for respiratory


depression and therefor should be
seen first. (A) Risk of hypotension.
(B) Lowest risk. (D) Risk of
nephrotoxicity and phlebitis.

dyspnea

ECG

shortness of breath; difficulty


breathing that may be caused by
certain heart conditions, strenuous
exercise, or anxiety.

U wave is positive deflectionfollowing the


T wave often present in hypokalemia
tall spiked T wave, prolonged QT
intervial, widening QRS complex are all
signs of hyperkalemia

elastic recoil

emphysema

empyema

the tendency for the lungs to recoil or


reduce in volume after being
stretched or expanded.

an abnormal condition of the


pulmonary system, characterized by
overinflation and destructive changes
in alveolar walls.

an accumulation of purulent
exudates in a body cavity, especially
the pleural space, as a result of
bacterial infection, such as pleurisy
or tuberculosis.

Endoplasmic Reticulum

Part of the cell where formation of


proteins occurs.

epistaxis

nosebleed

esophageal speech

a method of swallowing air, trapping


it in the esophagus, and releasing it
to create sound.

esophagogastromy

An excess of carbon dioxide in the blood causes an


increased respiratory rate and volume because CO2
A. displaces oxygen on hemoglobin, leading to a
decreased PaO2.
B. causes an increase in the amount of hydrogen ions
available in the body.
C. combines with water to form carbonic acid, lowering
the pH of cerebrospinal fluid.
D. directly stimulates chemoreceptors in the medulla to
increase respiratory rate and volume.

esophageal cancer
risk for infection = meticulious oral
care should be provided several times
a day prior to surgery

C. combines with water to form carbonic acid, lowering


the pH of cerebrospinal fluid. A combination of excess
CO2 and H2O results in carbonic acid, which lowers the
pH of the cerebrospinal fluid and stimulates an increase
in the respiratory rate. Peripheral chemoreceptors in the
carotid and aortic bodies also respond to increases in
PaCO2 to stimulate the respiratory center. Excess CO2
does not increase the amount of hydrogen ions available
in the body but does combine with the hydrogen of water
to form an acid.

A family member was taught to suction a client's


tracheostomy prior to the client's discharge from the
hospital. Which observation by the nurse indicates that
the family member is capable of correctly performing the
suctioning technique?
A. Turns on the continuous wall suction to -190 mm Hg
B. Inserts the catheter until resistance or coughing occurs
C. Withdraws the catheter while maintaining suctioning
D. Re-clears the tracheostomy after suctioning the mouth
A female client with a nasogastric tube attached to low
suction states that she is nauseated. The nurse assesses
that there has been no drainage through the nasogastric
tube in the last 2 hours. Which action should the nurse
take first?
A. Irrigate the nasogastric tube with sterile normal saline.
B. Reposition the client on her side.
C. Advance the nasogastric tube 5 cm.
D. Administer an intravenous antiemetic as prescribed.

flail chest

B. indicates correct technique for


performing suctioning. Suction pressure
should be between -80 and -120 (A). The
catheter should be withdrawn 1-2 cm at a
time with intermittent suction (C). (D)
introduces pathogens.
B. The priority is to determined if the
tube is functioning correctly, which
would relieve the client's nausea. The
least invasive intervention is to reposition
the client (B), should be attempted first,
followed by (A & C) if these are
unsuccessful then (D).

instability of the chest wall resulting


from multiple rib fractures.

Following a patient's bone marrow aspiration, which


of the following nursing interventions should a nurse
anticipate?
A. Application of firm pressure to the site B.
Positioning the patient in a prone position
C. Positioning the patient in a supine position
D. Application of a warm, moist compress to the site

A. Application of firm pressure to the site After a


bone marrow aspiration procedure, a nurse should
apply pressure to the aspiration site until bleeding
stops. Application of a warm, moist compress will not
alter the potential for bleeding. Positioning the
patient to assume a supine or prone position also will
not address the need to control bleeding from the
aspiration site.

Following a supraglottic laryngectomy, the patient is taught how to


use the supraglottic swallow to minimize the risk of aspiration. In
teaching the patient about this technique, the nurse instructs the
patient to
A. perform Valsalva maneuver immediately after swallowing.
B. breathe between each Valsalva maneuver and cough sequence.
C. cough after swallowing to remove food from the top of the vocal
cords.
D. practice swallowing thin, watery fluids before attempting to
swallow solid foods.

C. cough after swallowing to remove food from the top of


the vocal cords. A supraglottic laryngectomy involves
removal of the epiglottis and false vocal cords, and the
removal of the epiglottis allows food to enter the trachea.
Supraglottic swallowing requires performance of the
Valsalva maneuver before placing food in the mouth and
swallowing. The patient then coughs to remove food from
the top of the vocal cords, swallows again, and then
breathes after the food has been removed from the vocal
cords.

fremitus

vibration of the chest wall produced


by vocalization.

gangrene

necrosis/tissue death
priority prevent infection

hemothorax

accumulation of blood in the pleural


space.

Hepatitis B

Histologist

health care providers should have


Hep B vaccine;
transmitted by fecal/oral
contamination

Person who studies tissues.

The home health nurse is assessing a male client being treated for
Parkinson disease with levodopa-carbidopa (Sinemet). The nurse
observes that he does not demonstrate any apparent emotions
when speaking and rarely blinks. Which intervention should the
nurse implement?
A. Perform a complete cranial nerve assessment.
B. Instruct the client that he may be experiencing medication
toxicity.
C. Document the presence of these assessment findings.
D. Advise the client to seek immediate medical evaluation.

hospital-acquired pneumonia

hypercalcemia

hypercapnia

Hyperkalemia

C. A mask-like expression and infrequent


blinking are common clinical features of
Parkinsonism. The nurse should
document the findings. (A & D) are not
necessary. Signs of toxicity (B) are
dyskinesia, hallucinations, and psychosis.

pneumonia occurring 48 hours or


longer after hospital admission and
not incubating at the time of
hospitalization.

positive trousseau sign = carpal


spasm

greater than normal amounts of


carbon dioxide in the blood (PaCO2
> 45 mm Hg); also called
hypercarbia.

ECG=tall spiked T wave, prolonged


QT intervial, widening QRS complex
are all signs of hyperkalemia;
tumor lysis syndrome

hyperreactivity

hyperresponsiveness

an abnormal condition in which


responses to stimuli are exaggerated.

excessive or exaggerated response to


a stimulus; in asthma leads to
bronchoconstriction in response to
physical, chemical, or pharmacologic
stimuli.

Hypertrophy

Excessive devolopment.

hypocapnia

low arterial carbon dioxide pressure


(PaCO2 < 35 mm Hg); also called
hypocarbia.

Hypochondriac regions

The upper lateral regions of the


abdomen, beneath the ribs.

Hypoglossal (12th crainal nerve)

hypokalemia

hypomagnesemia

difficulty chewing, talking and


swallowing

patients on diuretics
will change patients normal ECG = U
wave is positive deflection following
the T wave often present in
hypokalemia

chronic alcholic
ie HR 160 BP 90/54 give IV
magnesium sulphate
prolonged QT intervial

If a health care provider is planning to transfuse a


patient with a unit of packed red blood cells, which of
the following solutions should the health care
provider hang with the transfusion?
A. 5% dextrose in water
B. 0.9% sodium chloride
C. 5% dextrose in 0.9% sodium chloride D. 5%
dextrose in lactated Ringer's solution

B. 0.9% sodium chloride The only


solution appropriate for administration
with whole blood or blood products is
0.9% sodium chloride. The other options
are not appropriate for use with blood
products.

If a nurse is assessing a patient whose recent blood gas


determination indicated a pH of 7.32 and respirations are
measured at 32 breaths/min, which of the following is the most
appropriate nursing assessment?
A. The rapid breathing is causing the low pH.
B. The nurse should sedate the patient to slow down respirations.
C. The rapid breathing is an attempt to compensate for the low pH.
D. The nurse should give the patient a paper bag to breathe into to
correct the low pH.

C. The rapid breathing is an attempt to compensate


for the low pH. The respiratory system influences pH
(acidity) through control of carbon dioxide
exhalation. Thus, rapid breathing increases the pH.
Breathing into a paper bag aids a patient who is
hyperventilating; in respiratory alkalosis, it aids in
lowering the pH. The use of sedation can cause
respiratory depression and hypoventilation, resulting
in an even lower pH.

If a nurse is caring for an 80-year-old patient with a


temperature of 100.4 F, crackles at the right lung
base, pain with deep inspiration, and dyspnea, which
of the following orders is the nurse's priority? A.
Sputum specimen for culture and sensitivity
B. Codeine 15 mg orally every 6 hours as needed
C. Incentive spirometer every 2 hours while awake
D. Amoxicillin (Amoxil) 500 mg orally 4 times a day

A. Sputum specimen for culture and sensitivity The patient presents


with signs of a respiratory infection. To initiate the most effective
therapy, the health care prescriber must know the pathogen
causing the infection. Therefore, the sputum specimen is the
nurse's priority. If the antibiotic is administered before the
specimen is obtained, the results of the culture might not be as
accurate and could impair the effectiveness of therapy. After the
specimen is obtained, the nurse can administer codeine for
coughing and begin the incentive spirometry to mobilize secretions
and improve the patient's ability to expectorate the secretions.

If a patient has pernicious anemia, the nurse


should provide information regarding
A. frequent bouts of dyspnea.
B. risks relative to dehydration.
C. deficiency of intrinsic factor.
D. lack of any effective treatment for this
condition.

C. deficiency of intrinsic factor.


Pernicious anemia is a type of anemia
caused by failure of absorption of vitamin
B12 (cobalamin). The most common
cause is lack of intrinsic factor, a
glucoprotein produced by the parietal
cells of the gastric lining.

If a patient states, "It's hard for me to breathe and I


feel short-winded all the time," what is the most
appropriate terminology to be applied in
documenting this assessment by a nurse?
A. Apnea
B. Dyspnea
C. Tachypnea
D. Respiratory fatigue

B. Dyspnea Dyspnea is a subjective description reflective


of the patient's statement indicating difficulty in
breathing. Apnea refers to absence of breath or breathing.
Tachypnea refers to an increased rate of breathing,
usually greater than 20 breaths per minute. Respiratory
fatigue is subjective and usually refers to the patient
exhibiting signs and symptoms associated with a
comprehensive respiratory assessment including laborious
breathing, use of accessory muscles, and slowing of
respirations.

If a patient with an uncuffed tracheostomy tube coughs


violently during suctioning and dislodges the
tracheostomy tube, a nurse should first
A. call the physician.
B. attempt to reinsert the tracheostomy tube.
C. position the patient in a lateral position with the neck
extended.
D. cover the stoma with a sterile dressing and ventilate
the patient with a manual bag-mask until the physician
arrives.

B. attempt to reinsert the tracheostomy


tube.Retention sutures may be grasped (if present)
and the tracheostomy opening spread, or a hemostat
may be used to spread the opening. The obturator is
inserted into the replacement tube (one size smaller
than the original tube), lubricated with saline
solution, and inserted into the stoma at a 45-degree
angle to the neck. If the attempt is successful, the
obturator tube should immediately be removed.

If a patient with arthritis develops irondeficiency anemia, a nurse should ask


about the patient's use of
A. alcoholic beverages.
B. stool softeners and laxatives.
C. caffeinated foods and beverages.
D. NSAIDs.

D. NSAIDs. NSAIDs decrease the level of vitamin C, which aids in


the absorption of iron. These drugs also compete with folate and
vitamin K and may cause gastritis. Excessive alcoholic beverage
consumption can cause stomach irritation; alcohol would not be
directly related to iron-deficiency anemia unless bleeding ulcers or
gastritis were to occur. NSAID consumption, not stool softeners
and laxative use, would be suspect for iron-deficiency anemia.
Caffeinated foods and beverages can cause gastric irritation and
discomfort but are not associated with iron-deficiency anemia.

If a patient with blood type O Rh- is given AB Rh- blood,


the nurse would expect
A. the patient's Rh factor to react with the RBCs of the
donor blood.
B. no adverse reaction because the patient has no
antibodies against the donor blood.
C. the anti-A and anti-B antibodies in the patient's blood
to hemolyze the donor blood.
D. the anti-A and anti-B antibodies in the donor blood to
hemolyze the patient's blood.

C. the anti-A and anti-B antibodies in the patient's blood to


hemolyze the donor blood. A patient with O Rh+ blood has no A or
B antigens on the red cell but does have anti-A and anti-B
antibodies in the blood and has an Rh antigen. AB Rh- blood has
both A and B antigens on the red cell but no Rh antigen and no
anti-A or anti-B antibodies. If the AB Rh- blood is given to the
patient with O Rh+ blood, the antibodies in the patient's blood will
react with the antigens in the donor blood, causing hemolysis of
the donor cells. There will be no Rh reaction because the donor
blood has no Rh antigen.

A. Observe for tiredness at the end of the day.


B. Perform a neurologic exam and mental status
exam.
C. Monitor for medication side effects.
D. Assess for decreased gross motor movement.

A. Sundowning syndrome is a pattern of


agitated behavior in the evening, believed
to be associated with tiredness at the end
of the day combined with fewer orienting
stimuli, such as activities and
interactions. (B, C, & D) with not provide
information about this syndrome.

In older adults, infection after exposure to respiratory illness is


most likely to
A. result in similar rates of infection as in the younger adult.
B. be easily prevented with the use of antibiotics after being
exposed.
C. result in serious lower respiratory infection related to weakened
respiratory muscles and fewer cilia.
D. be less serious because the older adult has less contact with
younger children who are most likely to carry serious infections.

C. result in serious lower respiratory infection


related to weakened respiratory muscles and fewer
cilia. Changes in the older adult respiratory system
make older adults more susceptible to infections that
can be very serious and life threatening. Use of
antibiotics to "prevent" lung infections is not
recommended and is ineffective for viral infections.

In preparing the preoperative teaching plan for a


patient who is to undergo a total laryngectomy, a
nurse should give highest priority to the
A. tracheostomy being in place for 2 to 3 days.
B. patient's not being able to speak normally again.
C. insertion of a gastrostomy feeding tube during
surgery.
D. patient's not being able to perform deep-breathing
exercises.

B. patient's not being able to speak normally again. Patients who


have a total laryngectomy have a permanent tracheostomy and will
need to learn how to speak using alternative methods, such as an
artificial larynx. The tracheostomy will be permanent to allow
normal breathing patterns and air exchange. After surgery, the
patient's nutrition is supplemented with enteral feedings, and
when the patient can swallow secretions, oral feedings can begin.
Deep-breathing exercises should be performed with the patient at
least every 2 hours to prevent further pulmonary complications.

In teaching the patient with COPD about the need for physical
exercise, the nurse informs the patient that
A. all patients with COPD should be able to increase walking
gradually up to 20 min/day.
B. a bronchodilator inhaler should be used to relieve exerciseinduced dyspnea immediately after exercise.
C. shortness of breath is expected during exercise but should return
to baseline within 5 minutes after the exercise.
D. monitoring the heart rate before and after exercise is the best
way to determine how much exercise can be tolerated.

C. shortness of breath is expected during exercise but should return


to baseline within 5 minutes after the exercise.Shortness of breath
usually increases during exercise, but the activity is not being
overdone if breathing returns to baseline within 5 minutes after
stopping. Bronchodilators can be administered 10 minutes before
exercise but should not be administered for at least 5 minutes after
activity to allow recovery. Patients are encouraged to walk 15 to 20
minutes a day with gradual increases, but actual patterns will
depend on patient tolerance. Dyspnea most frequently limits
exercise and is a better indication of exercise tolerance than is
heart rate in the patient with COPD.

In assessing an older client with dementia for


sundowning syndrome, what assessment technique is
best for the nurse to use?

In the case of pulmonary embolus from deep vein


thrombosis, which of the following actions should the
nurse take first?
A. Notify the physician.
B. Administer a nitroglycerin tablet sublingually.
C. Conduct a thorough assessment of the chest pain.
D. Sit the patient up in bed as tolerated and apply
oxygen.

D. Sit the patient up in bed as tolerated and


apply oxygen.The patient's clinical picture is
consistent with pulmonary embolus, and the
first action the nurse takes should be to
assist the patient. For this reason, the nurse
should sit the patient up as tolerated and
apply oxygen before notifying the physician.

Ischemia

Blood is held back from an area.

IV's infusions potential problems

jaundice

Karyotype

morphine, continous epidural =


respiratory depression
magnesium continous infusion =
hypotension
vancomycin intermittent infusion =
nephrotoxicity & phlebitits
serium amylase & lipase 2 times
higher than normal indicate
pancreatic injury
frothy tea colored urine
clay colored stools
complaints of puritis

Picture of nuclear structures arranged


in numerical order.

kidney stone

lactulose (Cephulac)

strain all urine most important


encourage urine

reduce blood ammonia by excreation


of ammonia by stools
2 -3 soft stools per day

Laryngectomy

cuff should be inflated only prior to


feeding

Laryngectomy

Removal of the voice box.

Larynx

Structure in the trachea.

Leukocyte

levodopa (Sinemet)

Eosinophil is a (an)

Parkinsons Disease
lessen tremors
increases amount of levodopa to CNS
(dopamine to the brain)
s/s toxicity=dyskinesia,
hallucinations, psychosis

Liver

RUQ contain the _______.

lung abscess

a pus-containing lesion of the lung


parenchyma that results in a cavity
formed by necrosis of lung tissue.

Lymphocyte

A blood cell that produces antibodies.

Magnesium Sulphate

A male client who has never smoked but has had


COPD for the past 5 years is now being assessed for
cancer of the lung. The nurse knows that he is most
likely to develop which type of lung cancer?
A. Adenocarcinoma
B. Oat-cell carcinoma
C. Malignant melanoma
D. Squamous-cell carcinoma
A male client with arterial peripheral vascular
disease (PVD) complains of pain in his feet. Which
instruction should the nurse give to the UPA to
quickly relieve the client's pain?
A. Help the client to dangle his legs.
B. Apply compression stockings.
C. Assist with passive leg exercises.
D. Ambulate three times daily.

Malignant myeloma

mechanical receptors

hypomagnesemia
reccomended for torsadesde pointes a
form of polymorphic ventrical
tackycardia associated with a
prolonged QT intervial that occurs
with hypomagnesemia
A. is the only lung cancer not related
to cigarette smoking related to lung
scarring and fibrosis from preexisting
pulmonary diseases such as TB and
COPD. (B& D) are related to
smoking. (C) is a skin cancer

A. A client who has arterial PVD may benefit from a


dependent position which can be achieved by
dangling by improving blood flow and relieving pain.
(B) is indicated for venous insufficiency and (C) is
indicated for bed rest. (D) is indicated to facilitate
collateral circulation and may improve long term
complaints of pain.

Tumor of bone marrow.

receptors located in lungs, upper


airways, chest wall, and diaphragm
that are stimulated by irritants,
muscle stretching, and alveolar wall
distortion.

Mediastinum

The space in the chest between the


lungs.

meniere syndrome (8th crainal


nerve)

tinnitus, vertigo, eharing difficulties

Metabolism

Sum of the chemical processes in a


cell.

metered-dose inhaler

aerosolized drug delivered in a


specific amount by activating the
inhaler or by inhaling.

methotrexate (Mexate)

immunosuppressant
can cause bone marrow depression
rheumatoid arthritis
lab=hemaglobin decrease =adverse
side effect

Mitochondria

Part of a cell where catabolism


primarily occurs.

The most appropriate position to assist a patient with chronic


obstructive pulmonary disease (COPD) who is having difficulty
breathing would be a
A. high Fowler's position without a pillow behind the head.
B. semi-Fowler's position with a single pillow behind the head.
C. right side-lying position with the head of the bed at 45 degrees'
elevation.
D. sitting upright and forward position with arms supported on an
over-the-bed table.

D. sitting upright and forward position with arms


supported on an over-the-bed table.Sitting upright
and leaning forward with arms supported on an overthe-bed table would be of most help to this patient,
because it allows for expansion of the thoracic cage in
all four directions (front, back, and two sides).

Myelogram

X-ray record of the spinal cord.

nadolol (Corgard)

nasal polyps

beta blocker
dibilatating anginal pain
bypass surgery patients
use with diuretic for hypertension
use extreme caution with respiratory
problems (asthma) and congestive heart
failure

benign mucous membrane masses


that form slowly in response to
repeated inflammation of the sinus or
nasal mucosa and project into the
nasal cavity.

Necr/o

Death

neuro function

altered neuro function =

neurogenic bladder

infection - from stasis of urine and


subsuquent catheterization

NG Tube

no drainage in 2 hours
client nausated = reposition client on
side

normocapnia

normal arterial carbon dioxide


pressure (PaCO2 35 to 45 mm Hg).

The nurse assesses a patient with


shortness of breath for evidence of longstanding hypoxemia by inspecting:
A. Chest excursion
B. Spinal curvatures
C. The respiratory pattern
D. The fingernail and its base
The nurse assesses a postoperative client. Oxygen is being
administered at 2 L/min and a saline lock is in place.
Assessment shows cool, pale, moist skin. The client is very
restless and has scant urine in the urinary drainage bag.
What intervention should the nurse implement first.
A. Measure urine specific gravity.
B. Obtain IV fluids for infusion protocol.
C. Prepare for insertion of a central venous catheter.
D. Auscultate the client's breath sounds.

D. The fingernail and its base Clubbing, a


sign of long-standing hypoxemia, is
evidenced by an increase in the angle
between the base of the nail and the
fingernail to 180 degrees or more, usually
accompanied by an increase in the depth,
bulk, and sponginess of the end of the finger.

B. The client is at risk for


hypovolemic shock and is exhibiting
early signs. Start IV to restore tissue
perfusion. (A, C, D) are all important
but less of a priority.

The nurse determines that a patient is experiencing


common adverse effects from the inhaled
corticosteroid beclomethasone (Beclovent) after
noting which of the following?
A. Adrenocortical dysfunction and hyperglycemia
B. Elevation of blood glucose and calcium levels
C. Oropharyngeal candidiasis and hoarseness
D. Hypertension and pulmonary edema

C. Oropharyngeal candidiasis and hoarseness


Oropharyngeal candidiasis and hoarseness are
common adverse effects from the use of inhaled
corticosteroids because the medication can lead to
overgrowth of organisms and local irritation if the
patient does not rinse the mouth following each dose.

The nurse determines that the patient is not


experiencing adverse effects of albuterol (Proventil)
after noting which of the following patient vital
signs?
A. Oxygen saturation 96%
B. Respiratory rate of 18
C. Temperature of 98.4 F
D. Pulse rate of 76

D. Pulse rate of 76 Albuterol is a 2agonist that can sometimes cause adverse


cardiovascular effects. These would
include tachycardia and angina. A pulse
rate of 76 indicates that the patient did
not experience tachycardia as an adverse
effect.

The nurse determines that the patient understood medication


instructions about the use of a spacer device when taking inhaled
medications after hearing the patient state which of the following
as the primary benefit?
A. "Now I will not need to breathe in as deeply when taking the
inhaler medications."
B. "This device will make it so much easier and faster to take my
inhaled medications."
C. "I will pay less for medication because it will last longer."
D. "More of the medication will get down into my lungs to help my
breathing."

D. "More of the medication will get


down into my lungs to help my
breathing." A spacer assists more
medication to reach the lungs, with
less being deposited in the mouth
and the back of the throat.

A nurse establishes the presence of a tension


pneumothorax when assessment findings reveal a(n)
A. absence of lung sounds on the affected side.
B. inability to auscultate tracheal breath sounds.
C. deviation of the trachea toward the side opposite
the pneumothorax.
D. shift of the point of maximal impulse (PMI) to the
left, with bounding pulses.

C. deviation of the trachea toward the side opposite


the pneumothorax. Tension pneumothorax is caused
by rapid accumulation of air in the pleural space,
causing severely high intrapleural pressure. This
results in collapse of the lung, and the mediastinum
shifts toward the unaffected side, which is
subsequently compressed.

The nurse evaluates that a patient is experiencing the


expected beneficial effects of ipratropium (Atrovent)
after noting which of the following assessment
findings?
A. Increased peak flow readings
B. Increased level of consciousness
C. Decreased sputum production
D. Increased respiratory rate

A. Increased peak flow readings.


Ipratropium is a bronchodilator that
should lead to increased PEFRs.

The nurse evaluates that discharge teaching for a patient hospitalized


with pneumonia has been most effective when the patient states which
of the following measures to prevent a relapse?
A. "I will increase my food intake to 2400 calories a day to keep my
immune system well."
B. "I must use home oxygen therapy for 3 months and then will have a
chest x-ray to reevaluate."
C. "I will seek immediate medical treatment for any upper respiratory
infections."
D. "I should continue to do deep-breathing and coughing exercises for at
least 6 weeks."

D. "I should continue to do deep-breathing and coughing


exercises for at least 6 weeks." It is important for the
patient to continue with coughing and deep breathing
exercises for 6 to 8 weeks until all of the infection has
cleared from the lungs. A patient should seek medical
treatment for upper respiratory infections that persist for
more than 7 days. Increased fluid intake, not caloric
intake, is required to liquefy secretions. Home O2 is not a
requirement unless the patient's oxygenation saturation is
below normal.

The nurse evaluates that nursing interventions to


promote airway clearance in a patient admitted with
COPD are successful based on which of the following
findings?
A. Absence of dyspnea
B. Improved mental status
C. Effective and productive coughing
D. PaO2 within normal range for the patient

C. Effective and productive coughing


The issue of the question is airway
clearance, which is most directly
evaluated as successful if the patient
can engage in effective and
productive coughing.

The nurse evaluates that teaching for the patient with


iron deficiency anemia has been effective when the
patient states A. "I will need to take the iron supplements
the rest of my life."
B. "I will increase my dietary intake of milk and milk
products."
C. "I should increase my activity to increase my aerobic
capacity."
D. "I should take the iron for several months after my
blood is normal."

D. "I should take the iron for several months after my


blood is normal." To replace the body's iron stores, iron
supplements should be continued for 2 to 3 months after
the Hb level returns to normal, but if the cause of the iron
deficiency is corrected, the supplements do not need to be
taken for a lifetime. Milk and milk products are poor
sources of dietary iron. Activity should be gradually
increased as Hb levels return to normal because aerobic
capacity can be increased when adequate Hb is available.

The nurse identifies the nursing diagnosis of activity


intolerance for a patient with asthma. The nurse
assesses for which of the following etiologic factor for
this nursing diagnosis in patients with asthma?
A. Anxiety and restlessness
B. Effects of medications
C. Fear of suffocation
D. Work of breathing

The nurse is assessing a 75-year-old male client for


symptoms of hyperglycemia. Which symptom of
hyperglycemia is an older adult most likely to
exhibit?
A. Polyuria
B. Polydipsia
C. Weight loss
D. Infection
The nurse is assessing a client who presents with
jaundice. Which assessment finding is the most
significant indication that further follow up is needed?
A. Urine specific gravity of 1.03 with a urine output of
500 ml in 8 hours
B. Frothy, tea-colored urine
C. Clay-colored stools and complaints of pruritus
D. Serum amylase and lipase levels that are twice their
normal levels

D. Work of breathingWhen the patient does


not have sufficient gas exchange to engage in
activity, the etiologic factor is often the work
of breathing. When patients with asthma do
not have effective respirations, they use all
available energy to breathe and have little
left over for purposeful activity.

D. S/Sx of hyperglycemia in older adults


may include fatigue, infection, and
neuropathy (such as sensory changes).
(A, B, C) are classic symptoms and may
be absent in the older adult.

D. Obstructive cholelithiasis and


alcoholism are the two major causes of
pancreatitis, and an elevated serum
amylase and lipase indicate pancreatic
injury. (A) is a normal finding. (B & C)
are expected findings for jaundice.

The nurse is assigned to care for a patient in the emergency


department admitted with an exacerbation of asthma. The patient
has received a -adrenergic bronchodilator and supplemental
oxygen. If the patient's condition does not improve, the nurse
should anticipate which of the following is likely to be the next step
in treatment?
A. Pulmonary function testing
B. Systemic corticosteroids
C. Biofeedback therapy
D. Intravenous fluids

B. Systemic corticosteroids Systemic


corticosteroids speed the resolution of
asthma exacerbations and are
indicated if the initial response to the
-adrenergic bronchodilator is
insufficient.

The nurse is assigned to care for a patient who has


anxiety and an exacerbation of asthma. Which of the
following is the primary reason for the nurse to
carefully inspect the chest wall of this patient?
A. Observe for signs of diaphoresis
B. Allow time to calm the patient
C. Monitor the patient for bilateral chest expansion
D. Evaluate the use of intercostal muscles

D. Evaluate the use of intercostal muscles


The nurse physically inspects the chest
wall to evaluate the use of intercostal
(accessory) muscles, which gives an
indication of the degree of respiratory
distress experienced by the patient.

The nurse is assisting a patient to learn self-administration of


beclomethasone two puffs inhalation q6hr. The nurse explains
that the best way to prevent oral infection while taking this
medication is to do which of the following as part of the selfadministration techniques?
A. Chew a hard candy before the first puff of medication.
B. Ask for a breath mint following the second puff of medication.
C. Rinse the mouth with water before each puff of medication.
D. Rinse the mouth with water following the second puff of
medication.

D. Rinse the mouth with water following


the second puff of medication. The
patient should rinse the mouth with water
following the second puff of medication
to reduce the risk of fungal overgrowth
and oral infection.

The nurse is caring for a 73-year-old patient who underwent a left


total knee arthroplasty. On the third postoperative day, the patient
complains of shortness of breath, slight chest pain, and that
"something is wrong." Temperature is 98.4o F, blood pressure
130/88, respirations 36, and oxygen saturation 91% on room air.
Which of the following should the nurse first suspect as the
etiology of this episode?
A. Septic embolus from the knee joint
B. Pulmonary embolus from deep vein thrombosis
C. New onset of angina pectoris
D. Pleural effusion related to positioning in the operating room

B. Pulmonary embolus from deep vein


thrombosis The patient presents the
classic symptoms of pulmonary embolus:
acute onset of symptoms, tachypnea,
shortness of breath, and chest pain.

The nurse is caring for a client with a chest tube to water


seal drainage that was inserted 10 days ago because of a
ruptured bullae and pneumothorax. Which finding
should the nurse report to the healthcare provider before
the chest tube is removed?
A. Tidal of water in the water seal chamber
B. Bilateral muffled breath sounds at bases
C. Temperature of 101 degrees F
D. Absence of chest tube drainage for 2 days.

A. Tidal in the water seal chamber should


be reported to the HPC to show that the
chest tube is working properly. (B) may
indicate hypoventilation from the chest
tube and usually improves when the tube
is removed. (C) indicates infection (D) is
an expected finding.

The nurse is caring for a critically ill client with cirrhosis


of the liver who has a nasogastric tube draining bright
red blood. The nurse notes that the client's serum
hemoglobin and hematocrit are decreased. What
additional change in lab data should the nurse expect?
A. Increased serum albumin
B. Decreased serum creatinine
C. Decreased serum ammonia
D. Increased liver function tests
The nurse is caring for a patient admitted to the hospital
with pneumonia. Upon assessment, the nurse notes a
temperature of 101.4 F, a productive cough with yellow
sputum and a respiratory rate of 20. Which of the
following nursing diagnosis is most appropriate based
upon this assessment? A. Hyperthermia related to
infectious illness
B. Ineffective thermoregulation related to chilling
C. Ineffective breathing pattern related to pneumonia
D. Ineffective airway clearance related to thick secretions

C. The breakdown of glutamine in the intestine and


the increased activity of colonic bacteria from the
digestion of proteins increases the ammonia levels in
the clients with advanced liver disease, so removal of
blood, a protein source, from the intestines results in
reduced ammonia. (A, B, D) will not be significantly
impacted by the removal of blood.

A. Hyperthermia related to infectious illness Because


the patient has spiked a temperature and has a
diagnosis of pneumonia, the logical nursing diagnosis
is hyperthermia related to infectious illness. There is
no evidence of a chill, and her breathing pattern is
within normal limits at 20 breaths per minute. There
is no evidence of ineffective airway clearance from
the information given because the patient is
expectorating sputum.

The nurse is caring for a patient who is to receive a


transfusion of two units of packed red blood cells. After
obtaining the first unit from the blood bank, the nurse
would ask which of the following health team members in
the nurses' station to assist in checking the unit before
administration?
A. Unit secretary
B. Another registered nurse
C. A physician's assistant
D. A phlebotomist

B. Another registered nurseBefore


hanging a transfusion, the registered
nurse must check the unit with
another RN or with a licensed
practical (vocational) nurse,
depending on agency policy.

The nurse is caring for a patient with an acute


exacerbation of asthma. Following initial treatment,
which of the following findings indicates to the nurse
that the patient's respiratory status is improving?
A. Wheezing becomes louder
B. Vesicular breath sounds decrease
C. Aerosol bronchodilators stimulate coughing
D. The cough remains nonproductive

A. Wheezing becomes louder The


primary problem during an exacerbation
of asthma is narrowing of the airway and
subsequent diminished air exchange. As
the airways begin to dilate, wheezing gets
louder because of better air exchange.

The nurse is caring for a patient with COPD and


pneumonia who has an order for arterial blood gases
to be drawn. Which of the following is the minimum
length of time the nurse should plan to hold pressure
on the puncture site?
A. 2 minutes
B. 5 minutes
C. 10 minutes
D. 15 minutes

B. 5 minutes Following obtaining an arterial blood


gas, the nurse should hold pressure on the puncture
site for 5 minutes by the clock to be sure that
bleeding has stopped. An artery is an elastic vessel
under higher pressure than veins, and significant
blood loss or hematoma formation could occur if the
time is insufficient.

The nurse is caring for a postoperative patient with sudden onset of


respiratory distress. The physician orders a STAT ventilationperfusion scan. Which of the following explanations should the
nurse provide to the patient about the procedure?
A. This test involves injection of a radioisotope to outline the blood
vessels in the lungs, followed by inhalation of a radioisotope gas.
B. This test will use special technology to examine cross sections of
the chest with use of a contrast dye.
C. This test will use magnetic fields to produce images of the lungs
and chest. D. This test involves injecting contrast dye into a blood
vessel to outline the blood vessels of the lungs.

A. This test involves injection of a radioisotope to


outline the blood vessels in the lungs, followed by
inhalation of a radioisotope gas.A ventilationperfusion scan has two parts. In the perfusion
portion, a radioisotope is injected into the blood and
the pulmonary vasculature is outlined. In the
ventilation part, the patient inhales a radioactive gas
that outlines the alveoli.

The nurse is completing an admission inter for a client with


Parkinson disease. Which question will provide addition
information about manifestations the client is likely to experience?
A. "Have you ever experienced and paralysis of your arms or legs?"
B. " Do you have frequent blackout spells?"
C. "Have you ever been 'frozen' in one spot, unable to move?"
D. "Do you have headaches, especially ones with throbbing pain?"

C. Parkinson clients frequently


experience difficulty in initiating,
maintaining, and performing motor
activities. They may even experience
being rooted, unable to move. (A, B, D)
Does not typically occur in Parkinson.

The nurse is evaluating whether a patient understands


how to safely determine whether a metered dose inhaler
is empty. The nurse interprets that the patient
understands this important information to prevent
medication underdosing when the patient describes
which method to check the inhaler?
A. Place it in water to see if it floats.
B. Shake the canister while holding it next to the ear.
C. Check the indicator line on the side of the canister.
D. Keep track of the number of inhalations used.
The nurse is interviewing a client who is taking
interferon-alfa-2a (Roferon-A) and ribavirin (Virazole)
combination therapy for hepatitis C. The client reports
experiencing overwhelming feelings of depression. What
action should the nurse implement first?
A. Recommend mental health counseling.
B. Review the medications actions and interactions.
C. Assess for the client's daily activity level.
D. Provide information regarding a support group.
The nurse is observing an unlicensed assistive personnel
(UPA) who is performing morning care for a bedfast
client with Huntington disease. Which care measure is
most important for the nurse to supervise?
A. Oral care
B. Bathing
C. Foot care
D. Catheter care

A nurse is performing assessment for a patient


diagnosed with chronic obstructive pulmonary
disease (COPD). Which of the following findings
should the nurse expect to observe?
A. Nonproductive cough
B. Prolonged inspiration
C. Vesicular breath sounds
D. Increased anterior-posterior chest diameter

The nurse is performing hourly neurological check for a


client with a head injury. Which new assessment finding
warrants the most immediate intervention by the nurse?
A. A unilateral pupil that is dilated and nonreactive to
light.
B. Client cries out when awakened by a verbal stimulus.
C. Client demonstrates a loss of memory to the events
leading up to the injury.
D. Onset of nausea, headache, and vertigo.

D. Keep track of the number of inhalations used. It is


no longer appropriate to see if a canister floats in
water or not as research has demonstrated this is not
accurate. The best method to determine when to
replace an inhaler is by knowing the maximum puffs
available per MDI and then replacing when those
inhalations have been used.

B. Alpha-interferon and ribavirin


combination therapy can cause
severe depression. (A, B, C) may be
implemented after physiological
aspect of the situation are assessed.

A. A client with Huntington disease


experiences problems with motor
skills such as swallowing and is at
high risk for aspiration. (B, C, D) do
not pose life-threatening
consequences.
D. Increased anterior-posterior chest diameter An
increased anterior-posterior diameter is a compensatory
mechanism experienced by patients with COPD and is
caused by air-trapping. Patients with COPD have a
productive cough, often expectorating copious amounts
of sputum. Because of air-trapping, patients with COPD
experience a prolonged expiration because the rate of gas
on exhalation takes longer to escape. Chest auscultation
for patients with COPD often reveals wheezing, crackles,
and other adventitious breath sounds.

A. Any changes in pupil size and


reactivity is an indication of increasing
ICP and should be reported immediately.
(B) is normal for being awakened. (C &
D) are common manifestations of head
injury and less of an immediacy than (A).

The nurse is planning the care for a client who is admitted with the
syndrome of inappropriate antidiuretic hormone secretion
(SIADH). Which interventions should the nurse include in this
client's plan of care? (Select all that apply.)
A. Salt-free diet
B. Quiet environment
C. Deep tendon reflex assessments
D. Neurologic checks
E. Daily weights
F. Unrestricted intake of free water

B, C, D, E.
SIADH results in water retention and dilutional hyponatremia,
which causes neurologic change when serum sodium levels are
less than 115 mEq/L. The nurse should maintain a quiet
environment (B) to prevent overstimulation that can lead to periods
of disorientation, assess deep tendon reflexes (C) and neurologic
checks (D) to monitor for neurologic deterioration. Daily weights
(E) should be monitored to assess for fluid overload: 1 kg weight
gain equals 1 L of fluid retention, which further dilutes serum
sodium levels. (A and F) contribute to dilutional hyponatremia.

A nurse is preparing to establish oxygen therapy for a patient with


COPD, and the physician's prescription reads "oxygen per nasal
cannula at 5 L per minute." Which of the following actions should
the nurse take?
A. Administer the oxygen as prescribed. B. Call the physician and
question the correct flow rate of the oxygen.
C. Establish the oxygen as prescribed and obtain an ABG.
D. Change the delivery device from a nasal cannula to a simple
oxygen mask.

B. Call the physician and question the correct flow rate of the oxygen.
The nurse should call the physician immediately and question the flow
rate for delivery of the oxygen before implementation. Oxygen is used
cautiously in patients with COPD because of longstanding hypoxemia
serving as the respiratory drive mechanism. If high levels of oxygen are
administered, the respiratory drive can be obliterated. Changing the
device to a simple oxygen mask may alter the oxygen concentration
being delivered to the patient and will further enhance the obliteration
of the patient's respiratory drive. Obtaining an ABG sample is not a
priority at this time, and the action does not address the validity of the
prescribed oxygen dosing for the patient.

A nurse is providing care to an adult female patient


and observes that the Hb laboratory analysis result is
9 g/dl. Based on this finding, the nurse should expect
to observe
A. dyspnea.
B. bradycardia.
C. warm, dry skin.
D. activity tolerance without complaint of fatigue.

A. dyspnea.Hb levels are used to determine the severity of anemia.


Patients with moderate anemia (Hb 6 to 10 g/dL) may suffer from
dyspnea, palpitations, diaphoresis with exertion, and chronic
fatigue. Patients who are anemic usually have cool skin related to
compensatory mechanism of mild vasoconstriction. Patients who
are anemic experience tachycardia because of increased demands
placed on the heart to meet overall metabolic requirements. Activity
tolerance without complaint is not correct because patients with
anemic conditions fatigue readily.

A nurse is reviewing the hematologic test results for


a patient in whom the hematocrit (Hct) is reported at
a reading of 30%. Based on this result, the nurse
should interpret that the patient
A. is susceptible to bleeding disorders.
B. has fewer red blood cells than normal. C. is
experiencing an inflammatory response.
D. is experiencing an acute hemolytic crisis.

B. has fewer red blood cells than normal.The Hct is the measure of
the volume of red blood cells in whole blood expressed as a
percentage. This test is useful in the diagnosis of anemia,
polycythemia, and abnormal hydration states. Patients who are
susceptible to bleeding disorders likely will have a low platelet
count. The inflammatory response may best be evaluated by
examination of results that include the white blood cell count with
differential analysis. Acute hemolytic crisis develops in patients
receiving blood components in which incompatibility occurs or in
patients with bleeding disorders or conditions that promote cellular
damage, such as damage associated with shock.

The nurse is reviewing the routine medications taken by a client


with chronic angle closure glaucoma. Which medication
prescription should the nurse question?
A. An antianginal with a therapeutic effect of vasodilation.
B. An anticholinergic with a side effect of pupillary dilation.
C. An antihistamine with a side effect of sedation.
D. A corticosteroid with a side effect of hyperglycemia.

B. Clients with angle closure glaucoma


should not take medications that dilate
the pupil (B) because this can precipitate
acute and severely increased intraocular
pressure. (A, C, D) do not cause
increased intraocular pressure, which is
the primary concern.

The nurse is scheduled to give a dose of ipratropium


bromide by metered dose inhaler. The nurse would
administer the right drug by selecting the inhaler
with which of the following trade names?
A. Vanceril
B. Pulmicort
C. AeroBid
D. Atrovent

D. Atrovent The trade or brand name


for ipratropium bromide, an
anticholinergic medication, is
Atrovent.

The nurse is scheduled to give a dose of salmeterol by


metered dose inhaler (MDI). The nurse would
administer the right drug by selecting the inhaler
with which of the following trade names?
A. Vanceril
B. Serevent
C. AeroBid
D. Atrovent

B. Serevent The trade or brand name


for salmeterol, an adrenergic
bronchodilator, is Serevent.

The nurse is teaching a patient how to self-administer ipratropium


(Atrovent) via a metered dose inhaler. Which of the following
instructions given by the nurse is most appropriate to help the
patient learn proper inhalation technique?
A. "Avoid shaking the inhaler before use." B. "Breathe out slowly
before positioning the inhaler."
C. "After taking a puff, hold the breath for 30 seconds before
exhaling."
D. "Using a spacer should be avoided for this type of medication."

B. "Breathe out slowly before positioning


the inhaler." It is important to breathe
out slowly before positioning the inhaler.
This allows the patient to take a deeper
breath while inhaling the medication thus
enhancing the effectiveness of the dose.

The nurse is teaching a patient who is to undergo bone marrow


aspiration. Which of the following statements made by the nurse
would indicate correct instruction regarding the site for the
aspiration procedure?
A. "The health care provider will perform the aspiration by needle
to the femur."
B. "The health care provider will perform the aspiration by needle
to the scapula." C. "The health care provider will perform the
aspiration by needle to the antecubital fossa."
D. "The health care provider will perform the aspiration by needle
to the posterior iliac crest."

D. "The health care provider will perform the


aspiration by needle to the posterior iliac crest."
Bone marrow samples are commonly taken from the
posterior iliac crest or, as an alternative, the sternum
may be aspirated. These sites provide relative ease in
accessing the bone marrow via the biopsy needle. The
antecubital fossa, femur, and scapula do not allow
access to bone marrow while also providing reduced
risk of harm to the patient.

A nurse is working on a respiratory care unit where many


of the patients are affected by asthma. Which of the
following actions by the nurse would most likely increase
respiratory difficulty for the patients?
A. Wearing perfume to work
B. Encouraging patients to ambulate daily C. Allowing
the patients to eat green leafy vegetables
D. Withholding antibiotic therapy until cultures are
obtained

A. Wearing perfume to work People with asthma should avoid extrinsic


allergens and irritants (e.g., dust, pollen, smoke, certain foods, colognes
and perfumes, certain types of medications) because their airways
become inflamed, producing shortness of breath, chest tightness, and
wheezing. Many green leafy vegetables are rich in vitamins, minerals,
and proteins, which incorporate healthy lifestyle patterns into the
patients' daily living routines. Routine exercise is a part of a prudent
lifestyle, and for patients with asthma the physical and psychosocial
effects of ambulation can incorporate feelings of well-being, strength,
and enhancement of physical endurance. Antibiotic therapy is always
initiated after cultures are obtained so that the sensitivity to the
organism can be readily identified.

The nurse know that a client taking diuretics must be


assessed for the development of hypokalemia, and that
hypokalemia will create changes in the client's normal
ECG tracing. Which ECG change would be an expected
finding in the client with hypokalemia?
A. Tall, spiked T waves
B. A prolonged QT interval
C. A widening QRS complex
D. Presence of a U wave

The nurse know that normal lab values expected for an adult may vary in an
older client. Which data would the nurse expect to find when reviewing
laboratory values of an 80-year-old man who is in good overall health.
A. Complet blood count reveals increased WBC and decreased RBC counts.
B. Chemistries reveal an increased serum bilirubin with slightly increased liver
enzymes.
C. Urinalysis reveals slight protein in the urine and bacteriuria with pyuria.
D. Serum electrolytes reveal a decreased sodium level with an increased
potassium level.

D. A U wave is a positive deflection


following the T wave and is often
present with hypokalemia. A, B, C
indicate hyperkalemia.

C. In older adults the protein found in


urine is slightly risen as a result of kidney
changes or subclinical UTIs and the
client frequently experiences
asymptomatic bacteriuria and pyuria as a
result of incomplete bladder emptying.
(A, B, D) are not normal findings.

The nurse notes a physician's order written at 10:00 AM


for 2 units of packed red blood cells to be administered to
a patient who is anemic secondary to chronic blood loss.
If the transfusion is picked up at 11:30, the nurse should
plan to hang the unit no later than which of the following
times?
A. 11:45 AM
B. 12:00 noon
C. 12:30 PM
D. 3:30 PM

B. 12:00 noon The nurse must hang


the unit of packed red blood cells
within 30 minutes of signing them
out from the blood bank

The nurse notices clear nasal drainage in a patient


newly admitted with facial trauma, including a nasal
fracture. The nurse should:
A. test the drainage for the presence of glucose.
B. suction the nose to maintain airway clearance.
C. document the findings and continue monitoring.
D. apply a drip pad and reassure the patient this is
normal.

A. test the drainage for the presence of


glucose. Clear nasal drainage suggests
leakage of cerebrospinal fluid (CSF). The
drainage should be tested for the
presence of glucose, which would
indicate the presence of CSF.

The nurse observes ventricular fibrillation on


telemetry and upon entering the clients bathroom
finds the client unconscious on the floor. What
intervention should the nurse implement first?

B. Ventricular fibrillation is a lifethreatening dysrhythmia and CPR


should be started immediately. A & C
are appropriate but B is the priority.
D does not address the seriousness of
the situation.

A. Administer an antidysrhythmic medication.


B. Start cardiopulmonary resuscitation.
C. Defibrillate the client at 200 joules.
D. Assess the client's pulse oximetry.

The nurse plans to help an 18-year-old developmentally disabled female client


ambulate on the first postoperative day. When the nurse tells her it is time to
get out of bed, the client becomes angry and yells at the nurse. "Get out of
here! I'll get up when I'm ready." Which response should the nurse provide?
A. "Your healthcare provider has prescribed ambulation on the first
postoperative day."
B. "You must ambulate to avoid serious complications that are much more
painful."
C. "I know how you feel; you're angry about having to do this, but it is
required."
D. "I'll be back in 30 minutes to help you get out of bed and walk around the
room."

D. Returning in 30 minutes provides a


cooling off period, is firm, direct,
nonthreatening, and avoids argument
with the client. B is threatening. C.
assumes what the client is feeling. A.
avoids the nurse's responsibility to
ambulate the client.

The nurse receives a physician's order to transfuse fresh frozen plasma to a


patient suffering from an acute blood loss. Which of the following procedures
is most appropriate for infusing this blood product?
A. Hand the fresh frozen plasma as a piggyback to a new bag of primary IV
solution without KCl.
B. Infuse the fresh frozen plasma as rapidly as the patient will tolerate.
C. Hang the fresh frozen plasma as a piggyback to the primary IV solution.
D. Infuse the fresh frozen plasma as a piggyback to a primary solution of
normal saline.

B. Infuse the fresh frozen plasma as rapidly as the


patient will tolerate. The fresh frozen plasma should
be administered as rapidly as possible and should be
used within 2 hours of thawing. Fresh frozen plasma
is infused using any straight-line infusion set. Any
existing IV should be interrupted while the fresh
frozen plasma is infused, unless a second IV line has
been started for the transfusion.

The nurse reviews pursed lip breathing with a patient newly diagnosed
with emphysema. The nurse reinforces that this technique will assist
respiration by which of the following mechanisms?
A. Preventing bronchial collapse and air trapping in the lungs during
exhalation
B. Increasing the respiratory rate and giving the patient control of
respiratory patterns
C. Loosening secretions so that they may be coughed up more easily
D. Promoting maximal inhalation for better oxygenation of the lungs

A. Preventing bronchial collapse and air


trapping in the lungs during exhalation
The focus of pursed lip breathing is to
slow down the exhalation phase of
respiration, which decreases bronchial
collapse and subsequent air trapping in
the lungs during exhalation.

The nurse who has administered a first dose of oral


prednisone (Deltasone) to the patient with asthma
writes on the care plan to begin monitoring which of
the following patient parameters?
A. Intake and output
B. Bowel sounds
C. Apical pulse
D. Deep tendon reflexes

A. Intake and output Corticosteroids


such as prednisone can lead to fluid
retention. For this reason, it is
important to monitor the patient's
intake and output.

The nurse witnesses a baseball player receive a blunt trauma to the back
of the head with a softball. What assessment data should the nurse
collect immediately?
A. Reactivity of deep tendon reflexes, comparing upper to lower
extremities.
B. Vital signs readings, excluding blood pressure if need equipment is
unavailable.
C. Memory of events that occurred before and after the blow to the head.
D. Ability to spontaneously open the eyes before any tactile stimuli are
given.

D. The LOC should be immediately established


immediately after the head injury has occurred.
Spontaneous eye opening (D) is a simple measure of
LOC. (A) is not the best indicator of LOC. (B) is
important but not the best indicator of LOC. (C) can
be assessed after LOC has been established by
assessing eye opening.

Nursing assessment findings of jugular vein


distention and pedal edema would be indicative of
which of the following complications of emphysema?
A. Acute respiratory failure
B. Pulmonary edema caused by left-sided heart
failure
C. Fluid volume excess secondary to cor pulmonale
D. Secondary respiratory infection

C. Fluid volume excess secondary to cor pulmonale


Cor pulmonale is a right-sided heart failure caused by
resistance to right ventricular outflow due to lung
disease. With failure of the right ventricle, the blood
emptying into the right atrium and ventricle would
be slowed, leading to jugular venous distention and
pedal edema.

Nursing interventions for the patient with


aplastic anemia are directed toward the
prevention of the complications of
A. fatigue and dyspnea.
B. hemorrhage and infection.
C. thromboemboli and gangrene.
D. cardiac arrhythmias and heart failure.

B. hemorrhage and infection. Hemorrhage


from thrombocytopenia and infection from
neutropenia are the greatest risks for the
patient with aplastic anemia. The patient
will experience fatigue from anemia, but
bleeding and infection are the major causes
of death in aplastic anemia.

O2 toxicity

obstructive sleep apnea

older adults

a condition of oxygen overdosage


caused by prolonged exposure to a
high levels of oxygen; may inactivate
pulmonary surfactant and lead to
development of acute respiratory
distress syndrome.

a condition characterized by partial


or complete upper airway obstruction
during sleep, causing apnea and
hypopnea.

stooped posture results in upper torso


becomming center of gravity

older adults

An older client is admitted with a diagnosis of


bacterial pneumonia. The nurse's assessment of the
client will most likely reveal which S/SX?
A. Leukocytosis and febrile.
B. Polycythemia and crackles.
C. Pharyngitis and sputum production.
D. Confusion and tachycardia.

An older female client with dementia is transferred from a long term


care unit to an acute care unit. The client's children express concern that
their mother's confusion is worsening. How should the nurse respond?
A. "It is to be expected that older people will experience progressive
confusion."
B. "Confusion in an older person often follows relocation to new
surroundings."
C. "The dementia is progressing rapidly, but we will do everything we
can to keep your mother safe."
D. "The acute care staff is not as experienced as the long-term care staff
at dealing with dementia."

An older male client comes to the geriatric screening clinic


complaining of pain in his left calf. The nurse notices a reddened
area on the calf of his right leg that is warm to touch and the nurse
suspects that the client may have thrombophlebitis. Which
addition assessment is most important for the nurse to perform?
A. Measure calf circumference.
B. Auscultate the client's breath sounds.
C. Observe for ecchymosis and petechiae.
D. Obtain the client's blood pressure.

olpthalmoscopic exam

protein found in urine slightly rises


as a result of kidney change or UTI
w/asymptomatic bacteriuria and
pyuria as a result of incomplete
bladder emptying

D. The onset of pneumonia is the older


may be signaled by general deterioration,
confusion, increased heart rate or
increased respiratory rate.
(A, B, C) are often absent in the older
with bacterial pneumonia.

B. Relocation often results in confusion


among older clients and is stressful to clients
of all ages. (A) is an inaccurate stereotype.
(C) is most likely false there are many factors
that cause increased temporary confusion.
(D) may be true but does not offer the family
a sense of security about the care.

B. Since the client may have a pulmonary


embolus secondary to the
thrombophlebitis.
A. Would support the nurses assessment.
C. Least helpful since bruising is not
associated with thrombophlebitis.
D. Less important then auscultation.

from a distance of 8-12 inches and slightly to the side,


shine the light in the clients pupil;
client should focus on distant object to promote
dialation, olpthalmoscopic set at 0 lens to begin,
should be held in front of the examiners left eye when
examining clients right eye and kept within 1 inch of
clients eye for optimum visualation

osteoporosis

most common cause of fractured hip=


reduced calcium in bones result of
hormonal changes during
perimenaopausal years

Otalgia

Pain in the ear.

pancreatic insufficiency

a condition characterized by
inadequate production and secretion
of pancreatic hormones or enzymes.

pancreatitis

panlobular emphysema

serium amylase & lipase 2 to 5 times


higher than normal
hypercalcemia 40 ~ 75% = positive
trousseau sign = carpal spasm
severe boring pain
type of emphysema involving distention
and destruction of the entire primary
respiratory lobule; usually associated
with "1-antitrypsin deficiency; also called
chronic hypertrophic, diffuse,
generalized, panlobular, or vesicular
emphysema.

Pap smear

A patient admitted to the emergency department


with tension pneumothorax and mediastinal shift
following an automobile crash is most likely to
exhibit A. bradycardia.
B. severe hypotension.
C. mediastinal flutter.
D. a sucking chest wound.

The patient has an order for albuterol 5 mg via


nebulizer. Available is a solution containing 1 mg/ml.
How many milliliters should the nurse use to prepare
the patient's dose?
A. 0.2
B. 2.5
C. 3.75
D. 5.0

The patient has an order for albuterol 5 mg via


nebulizer. Available is a solution containing 2 mg/ml.
How many milliliters should the nurse use to prepare
the patient's dose?
A. 0.2
B. 2.5
C. 3.75
D. 5.0

The patient has an order for each of the following


inhalers. Which of the following should the nurse
offer to the patient at the onset of an asthma attack?
A. Albuterol (Proventil)
B. Beclomethasone (Beclovent)
C. Ipratropium bromide (Atrovent)
D. Salmeterol (Serevent)

should be continued through


menapause to test for vaginal and
cervical cancer

B. severe hypotension. Mediastinal shift may cause


compression of the lung in the direction of the shift and
compression, traction, torsion, or kinking of the great
vessels. Blood return to the heart is dangerously impaired
and causes a subsequent decrease in cardiac output and
blood pressure. Tachycardia is a clinical manifestation of
tension pneumothorax. An uncovered opened
pneumothorax is associated with a sucking chest wound
and mediastinal flutter.

D. 5.0

B. 2.5

A. Albuterol (Proventil) Albuterol is a


short-acting bronchodilator that
should initially be given when the
patient experiences an asthma
attack.

A patient has been receiving oxygen per nasal cannula while


hospitalized for COPD. The patient asks the nurse whether oxygen
use will be needed at home. Which of the following would be the
most appropriate response by the nurse? A. "Long-term home
oxygen therapy should be used to prevent respiratory failure."
B. "Oxygen will be needed when your oxygen saturation drops to
88% and you have symptoms of hypoxia.
C. "Long-term home oxygen therapy should be used to prevent
heart problems related to emphysema."
D. "Oxygen will not be needed until or unless you are in the
terminal stages of this disease."

B. "Oxygen will be needed when your oxygen


saturation drops to 88% and you have symptoms of
hypoxia.Long-term oxygen therapy in the home
should be considered when the oxygen saturation is
88% or less and the patient has signs of tissue
hypoxia, such as cor pulmonale, erythrocytosis, or
impaired mental status.

A patient is admitted to the hospital with fever, chills, a


productive cough with rusty sputum, and pleuritic chest
pain. Pneumococcal pneumonia is suspected. An
appropriate nursing diagnosis for the patient based on
the patient's manifestations is
A. hyperthermia related to acute infectious process.
B. chronic pain related to ineffective pain management.
C. risk for injury related to disorientation and confusion.
D. ineffective airway clearance related to retained
secretions.

A. hyperthermia related to acute infectious process. The patient


with pneumococcal pneumonia is acutely ill with fever and the
systemic manifestations of fever, such as chills, thirst, headache,
and malaise. Interventions that monitor temperature and aid in
lowering body temperature are appropriate. Ineffective airway
clearance would be manifested by adventitious breath sounds and
difficulty producing secretions. Disorientation and confusion are
not noted in this patient and are not typical unless the patient is
very hypoxemic. Pleuritic pain is an acute pain that is due to
inflammation of the pleura.

A patient is being discharged from the emergency


department after being treated for epistaxis. In teaching
the family first aid measures in the event the epistaxis
would recur, which of the following measures would the
nurse suggest? (Select all that apply.)
A. Tilt patients head backwards
B. Apply ice compresses to the nose
C. Pinch the entire soft lower portion of the nose
D. Partially insert a small gauze pad into the bleeding
nostril

B,C,D First aid measures to control epistaxis includes


placing the patient in a sitting position, leaning
forward. Tilting the head back does not stop the
bleeding, but rather allows the blood to enter the
nasopharynx, which could result in aspiration or
nausea/vomiting from swallowing blood. All of the
other options are appropriate first aid treatment of
epistaxis.

A patient is having inspiratory stridor (crowing


respiration) and the nurse suspects he is experiencing a
laryngospasm. Which of the following would be most
appropriate to implement for a patient experiencing a
laryngospasm?
A. Administer 100% oxygen.
B. Position the patient in high Fowler's position.
C. Insert a 16-gauge (large-bore) IV needle.
D. Activate the emergency response team (code blue
team) to the patient's room.

A. Administer 100% oxygen.A nurse should immediately


administer 100% oxygen to the patient until the airway is fully
reestablished, the larynx relaxes, and the spasms stop. Activating
the emergency response team is not an immediate nursing action at
this time because the nurse can administer the oxygen without the
assistance of others. Positioning the patient in high Fowler's will
not address the patient's need for immediate reoxygenation because
of the patient's compromised respiratory state. Insertion of an IV
device is not the first priority response but should be implemented
after the nurse has assessed that the airway is stable.

A patient with acute exacerbation of COPD needs to


receive precise amounts of oxygen. Which of the
following types of equipment should the nurse
prepare to use?
A. Venturi mask
B. Partial non-rebreather mask
C. Oxygen tent
D. Nasal cannula

A. Venturi mask The Venturi mask


delivers precise concentrations of
oxygen and should be selected
whenever this is a priority concern.
The other methods are less precise in
terms of amount of oxygen delivered.

A patient with an acute pharyngitis is seen at the clinic


with fever and severe throat pain that affects swallowing.
On inspection the throat is reddened and edematous with
patchy yellow exudates. The nurse anticipates that
collaborative management will include
A. treatment with antibiotics.
B. treatment with antifungal agents.
C. a throat culture or rapid strep antigen test.
D. treatment with medication only if the pharyngitis does
not resolve in 3 to 4 days.

C. a throat culture or rapid strep antigen test.


Although inadequately treated -hemolytic
streptococcal infections may lead to rheumatic heart
disease or glomerulonephritis, antibiotic treatment is
not recommended until strep infections are definitely
diagnosed with culture or antigen tests. The
manifestations of viral and bacterial infections are
similar, and appearance is not diagnostic except when
candidiasis is present.

A patient with COPD is receiving oxygen at 2 L/min.


While in the supine position for a bath, the patient
complains of shortness of breath. What is the most
appropriate first nursing action?
A. Increase the flow of oxygen.
B. Perform tracheal suctioning.
C. Report this to the physician.
D. Assist the patient to Fowler's position.

D. Assist the patient to Fowler's position. Breathing


is easier in Fowler's position because it permits
greater expansion of the chest cavity. If repositioning
does not improve the situation, then oxygenation and
physician reporting might be appropriate. The
patient would not benefit from tracheal suctioning.

A patient with hemophilia is hospitalized with acute knee


pain and swelling. An appropriate nursing intervention
for the patient includes
A. wrapping the knee with an elastic bandage.
B. placing the patient on bed rest and applying ice to the
joint.
C. gently performing range-of-motion (ROM) exercises to
the knee to prevent adhesions.
D. administering nonsteroidal anti-inflammatory drugs
(NSAIDs) as needed for pain.

B. placing the patient on bed rest and applying ice to


the joint. During an acute bleeding episode in a joint,
it is important to totally rest the involved joint and
slow bleeding with application of ice. Drugs that
decrease platelet aggregation, such as aspirin or
NSAIDs, should not be used for pain. As soon as
bleeding stops, mobilization of the affected area is
encouraged with range-of-motion (ROM) exercises
and physical therapy.

A patient's ABGs include a PaO2 of 88 mm Hg and a


PaCO2 of 38 mm Hg and mixed venous blood gases
include a PvO2 of 40 mm Hg and PvCO2 of 46 mm Hg.
These findings indicate that the patient has
A. impaired cardiac output.
B. unstable hemodynamics.
C. inadequate delivery of oxygen to the tissues.
D. normal capillary oxygen-carbon dioxide exchange.

D. normal capillary oxygen-carbon dioxide exchange. Normal


venous blood gas values reflect the normal uptake of oxygen from
arterial blood and the release of carbon dioxide from cells into the
blood, resulting in a much lower PaO2 and an increased PaCO2.
The pH is also decreased in mixed venous blood gases because of
the higher PvCO2. Normal mixed venous blood gases also have
much lower PvO2 and SvO2 than arterial blood bases. Mixed
venous blood gases are used when patients are hemodynamically
unstable to evaluate the amount of oxygen delivered to the tissue
and the amount of oxygen consumed by the tissues.

pentobarbital (Neubatal sodium)

contriandicated with liver damage

permanent pacemaker

changes in pulse rate in rythem may indicate


pacer failure
dizziness may be due to decreased heart rate
leading to decreased cardiac output;
should carry a card in wallet with type and
serial number of pacemaker;
report redness and tenderness - s/s infection

A person complains of fatigue and malaise and has a slight temperature


elevation for 2 days before symptoms of influenza (fever, chest
congestion, and productive cough) become noticeable. During the time
immediately before the illness is diagnosed, the patient
A. could avoid contracting the disease if treatment is begun with
antibiotics.
B. is unable to spread the disease because it is still in the incubation
period. C. is in the prodromal stage and is highly contagious and able to
spread the disease.
D. has a nosocomial infection, which affects approximately two million
individuals a year.

C. is in the prodromal stage and is highly contagious and


able to spread the disease. The prodromal stage is a short
period of time (hours to several days) immediately
preceding the onset of an illness during which the patient
is very contagious. Antibiotics are not effective against
viral illnesses. The incubation period is the time from
entry of the organism to the onset of symptoms and, in
some viral illnesses, may be contagious. Nosocomial
infections are those acquired in a hospital, and this
scenario does not suggest the source of the infection.

Pharynx

Throat.

The physician has prescribed salmeterol (Serevent) for a patient with


asthma. In reviewing the use of dry powder inhalers (DPIs) with the
patient, the nurse should provide which of the following instructions?
A. "Close lips tightly around the mouthpiece and breathe in deeply and
quickly."
B. "To administer a DPI, you must use a spacer that holds the medicine
so that you can inhale it."
C. "Hold the inhaler several inches in front of your mouth and breathe
in slowly, holding the medicine as long as possible." D. "You will know
you have correctly used the DPI when you taste or sense the medicine
going into your lungs."

Pleural Cavity

A. "Close lips tightly around the mouthpiece and


breathe in deeply and quickly." Dry powder inhalers
do not require spacer devices. The patient should be
instructed to breathe in deeply and quickly to ensure
medicine moves down deeply into lungs. The patient
may not taste or sense the medicine going into the
lungs.

Space between the membranes


around the lungs.

pleural effusion

pleural friction rub

an abnormal accumulation of fluid in


the intrapleural spaces of the lungs.

creaking or grating sound from


roughened, inflamed surfaces of the
pleura rubbing together, evident during
inspiration, expiration, or both and no
change with coughing; usually
uncomfortable, especially on deep
inspiration.

pleurisy (pleuritis)

inflammation of the pleura.

pneumoconiosis

a general term for lung diseases


caused by inhalation and retention of
dust particles.

pneumonia

an acute inflammation of the lungs,


often caused by inhaled pneumococci
of the species Streptococcus
pneumoniae.

pneumothorax

postural drainage

pulmonary edema

pulmonary embolism

pulmonary hypertension

a collection of air or gas in the


pleural space causing the lung to
collapse.

the use of various positions to


promote gravity drainage of
bronchial secretions; coughing
usually expels secretions of the
trachea.
an abnormal accumulation of fluid in the
alveoli and interstitial spaces of the lungs
caused most commonly by heart failure;
an acute, life-threatening situation in
which the lung alveoli become filled with
serous or serosanguineous fluid caused
most commonly by heart failure.
a thromboembolic occlusion of the
pulmonary vasculature resulting from
thrombi in the venous circulation or right
side of the heart and from other sources,
such as amniotic fluid, air, fat, bone marrow,
and foreign IV material that travel as emboli
until lodging in the pulmonary vessels.

elevated pulmonary pressure


resulting from an increase in
pulmonary vascular resistance to
blood flow through small arteries and
arterioles.

pursed-lip breathing

a technique of exhaling against pursed


lips to prolong exhalation, preventing
bronchiolar collapse and air trapping;
done to increase expiratory airway
pressure, improve oxygenation of the
blood, and help prevent early airway
closure.

PVD peripheral vascular disease

help client dangle legs

Respiratory acidosis is at highest risk


in a patient with
A. hypokalemia.
B. pulmonary fibrosis.
C. salicylate overdose.
D. COPD.

D. COPD. Chronic respiratory acidosis is most


commonly caused by COPD. Pulmonary fibrosis,
hypokalemia, and salicylate overdose do not
predispose a patient to respiratory acidosis.
Hypokalemia can lead to cardiac dysrhythmias.
Salicylate overdose results in central nervous system
changes, and pulmonary fibrosis can result in
respiratory arrest.

The resurgence in TB resulting from the emergence


of multidrug-resistant strains of Mycobacterium
tuberculosis is primarily the result of
A. a lack of effective means to diagnose TB.
B. poor compliance with drug therapy in patients
with TB.
C. the increased population of immunosuppressed
individuals with AIDS. D. indiscriminate use of
antitubercular drugs in treatment of other infections.

B. poor compliance with drug therapy in patients with


TB. Drug-resistant strains of TB have developed because
TB patients' compliance to drug therapy has been poor
and there has been general decreased vigilance in
monitoring and follow-up of TB treatment.
Antitubercular drugs are almost exclusively used for TB
infections. TB can be effectively diagnosed with sputum
cultures. The incidence of TB is at epidemic proportions
in patients with HIV, but this does not account for drugresistant strains of TB.

rhinoplasty

the surgical reconstruction of the


nose.

rhonchi

continuous rumbling, snoring, or


rattling sounds from obstruction of
large airways with secretions; most
prominent on expiration; change
often evident after coughing or
suctioning.

Sagittal

A plane that divides the body into


right and left portions.

Sarcoma

Malignant tumor of flesh tissue.

Seconal 0.1 gram PRN at bedtime is prescribed for


rest. The scored tablets are labeled grain 1.5 per
tablet. How many tablets should the nurse plan to
administer?
A. 1/2 tablet
B. 1 tablet
C. 1 1/2 tablet
D. 2 tablets

seconel

B. 15 gr = 1 g, 0.1 x 15 = 1.5 grains

sleep aide
15g=1g

Select all that apply.


Atelectasis can be caused by
A. long-term smoking.
B. inadequate surfactant.
C. localized airway obstruction.
D. an increase in lung expansion.
E. an increase in elastic recoil.

BCE The collapse of lung tissue has


several causes, including reduced lung
expansion, localized airway obstruction,
inadequate surfactant, and an increase in
elastic recoil. Smoking, although
harmful, does not in itself cause
atelectasis.

Select all that apply. Which of the following are


significant risk factors for leukemia?
A. Being a longtime smoker
B. Employment in an oil refinery
C. History of hemophilia in parent
D. Having Down syndrome
E. Having a twin brother with leukemia
F. Treatment with an alkylating agent = 3 years ago

BDEF Exposure to chemical agents,


treatment with alkylating cancer
drugs, leukemia in a sibling, and the
patient's having Down syndrome are
all risk factors for leukemia.

SIADH inappropriate antidiuretic


hormone secreation

water retention & dilutional


hyponatremia
POC=quiet enviroment, deep tendon
reflex assessment, neurologic checks,
daily weights
1kg=1L

Skin cell

An epithelial cell is a (an)

small bowel obstruction

peritonis w/ Temperature of 102


notify HCP immediately
abdominal cramping

status asthmaticus

a severe, life-threatening asthma


attack that is refractory to usual
treatment and places the patient at
risk for developing respiratory failure.

sundowning

agitated behavior in the evening


observe for tirdness at the end of the
day

Supine

Lying on the back.

surfactant

telemetry

a lipoprotein that lowers the surface


tension in the alveoli, reduces the
amount of pressure needed to inflate
the alveoli, and decreases the
tendency of the alveoli to collapse.

ventrical fibrillation = life


threatening start CPR

tension pneumothorax

a pneumothorax with rapid


accumulation of air in the pleural
space causing severely high
intrapleural pressures with resultant
tension on the heart and great
vessels.

thoracentesis

a surgical procedure done to remove


fluid from the pleural space.

thoracotomy

surgical opening into the thoracic


cavity.

tidal volume

volume of air exchanged with each


breath.

To ensure the correct amount of oxygen delivery for a patient


receiving 35% oxygen via a Venturi mask, it is most important that
the nurse
A. keep the air-entrainment ports clean and unobstructed.
B. apply an adaptor to increase humidification of the oxygen.
C. drain moisture condensation from the oxygen tubing every hour.
D. keep the flow rate high enough to keep the bag from collapsing
during inspiration.

A. keep the air-entrainment ports clean and unobstructed. Oxygen is


delivered to a small jet in the center of a wide-based cone. Air is
entrained (pulled through) openings in the cone as oxygen flows through
the small jet. The degree of restriction or narrowness of the jet
determines the amount of entrainment and the dilution of pure oxygen
with room air and thus the concentration of oxygen. Although applying
an adaptor can increase the humidification with the Venturi mask, it is
not the best answer, because an open port is essential to proper
functioning. Draining moisture condensation from the oxygen tubing is
performed as often as needed, not on an hourly schedule. A plastic face
mask with a reservoir bag needs to have sufficient flow rate to keep the
bag inflated.

To find the infection site associated with


acute lymphangitis, the nurse should look
_____ to the inflammation.
A. distal
B. anterior
C. proximal
D. contralateral

A. distal The nurse should assess


distal to swelling to locate the initial
site of infection. Examining
proximal, contralateral, or anterior to
the inflammation does not describe
swelling associated with infection.

To prevent atelectasis in an 82-year-old


patient with a hip fracture, a nurse should
A. supply oxygen.
B. suction the upper airway.
C. ambulate the patient frequently.
D. assist the patient with aggressive
coughing and deep breathing.

D. assist the patient with aggressive coughing and


deep breathing. Decreased mobility after surgery in
older adults creates the possibility of fluid buildup
and retention in lung tissue. One of the primary goals
of nursing intervention is to prevent atelectasis in a
high-risk patient. Aggressive coughing and deep
breathing can prevent atelectasis in the postoperative
patient.

To promote airway clearance in a patient with


pneumonia, the nurse instructs the patient to do
which of the following? (Select all that apply.)
A. Splint the chest when coughing
B. Maintain a semi-Fowler's position
C. Maintain adequate fluid intake
D. Instruct patient to cough at end of exhalation

A,C,D The nurse should instruct the patient to splint


the chest while coughing. This will reduce discomfort
and allow for a more effective cough. Maintaining
adequate fluid intake liquefies secretions, allowing
easier expectoration. Coughing at the end of
exhalation promotes a more effective cough. The
patient should be positioned in an upright sitting
position (high-Fowler's) with head slightly flexed.

Tonsillitis

Inflammation of lymph tissue in the


throat.

TPN total protein nutrition

only regular insulin is adm. IV return


containing NPH

tracheostomy

tracheotomy

trigeminal neuralgia (5th crainal


nerve)

trigger

TSS

surgical opening into the trachea


through which an indwelling tube
may be inserted.

a surgical incision into the trachea


for the purpose of establishing an
airway; performed below a blockage
by a foreign body, tumor, or edema of
the glottis.

sudden stabbing severe pain over the


lip and chin

a substance, object, or agent that


initiates or stimulates an action; in
asthma, any stimuli that initiates the
IgE

Staphlococcus aures produce a toxin


that can enter the blood stream
through vaginal mucosa.
wash hands before and change
tampon frequently 4-6 hours

tuberculosis

tumor lysis syndrome

Twelve hours after chest tube insertion for hemothorax,


the nurse notes that the client's drainage has decreased
from 50 ml/hr to 5 ml/hr. What is the best inital action
for the nurse to take?
A. Document this expected decrease in drainage.
B. Clamp the chest tube while assessing for air leaks.
C. Milk the tube to remove any excessive blood clot build
up.
D. Assess for kinks or dependent loops in the tubing.

an infectious disease caused by


Mycobacterium tuberculosis; usually
involves the lungs but also occurs in the
larynx, kidneys, bones, adrenal glands,
lymph nodes, and meninges and can be
disseminated throughout the body.

hyperkalemia may occur =requires


insulin to reduce serum potassium =
monitor serium potassium and blood
glucose levels

D. The least invasive action should be performed to


assess the decrease in drainage. (A) is completed
after assessing for and problems causing the
decreased drainage. (B) is no longer protocol because
the increased pressure may be harmful for the client.
(C) is an appropriate nursing action after the tube has
been assessed for kinks or dependent loops.

Upon entering the room of a patient who has just returned from
surgery for total laryngectomy and radical neck dissection, a nurse
should recognize a need for intervention when finding
A. a gastrostomy tube that is clamped.
B. the patient coughing blood-tinged secretions from the
tracheostomy.
C. the patient positioned in a lateral position with the head of the
bed flat.
D. 200 ml of serosanguineous drainage in the patient's portable
drainage device.

C. the patient positioned in a lateral


position with the head of the bed flat.
After total laryngectomy and radical neck
dissection, a patient should be placed in a
semi-Fowler's position to decrease edema
and limit tension on the suture line.

Using light pressure with the index and middle fingers,


the nurse cannot palpate any of the patient's superficial
lymph nodes. The nurse
A. records this finding as normal.
B. should reassess the lymph nodes using deeper
pressure.
C. asks the patient about any history of any radiation
therapy.
D. notifies the health care provider that x-rays of the
nodes will be necessary.

A. records this finding as normal.


Superficial lymph nodes are evaluated by
light palpation, but they are not normally
palpable. It may be normal to find small
(<1.0 cm), mobile, firm, nontender
nodes. Deep lymph nodes are detected
radiographically.

vecuronium bromide (Norcuron)

vibration

Viscera

skeletal muscle relaxatant


ND: impaired communication R/T
paralysis of skeletal muscles

pressing on the chest with the flat of


the hands while repeatedly tensing
the hand and arm muscles to
facilitate movement of secretions to
larger airways.

Internal organs.

What is the correct location for the placement of the hand for
manual chest compressions during CPR on the adult client.
A. Just above the xiphoid process on the upper third of the
sternum.
B. Below the xiphoid process midway between the sternum and the
umbilicus.
C. Just about the xiphoid process on the lower third of the sternum.
D. Below the xiphoid process midway between the sternum and the
first rib.

What is the correct procedure for performing an ophthalmoscopic


examination on a client's right eye?
A. Instruct the client to look at the examiner's nose and not move
his/her eyes during the exam.
B. Set ophthalmoscope on the plus 2 to 3 lens and hold it in front of
the examiner's right eye.
C. From a distance of 8 to 12 inches and slightly to the side, shine
the light into the client's pupil.
D. For optimum visualization, keep the ophthalmoscope at least 3
inches for the client's eye

C.

C. The client should focus on a distant object in order


to promote pupil dilation. The ophthalmoscope
should be set on the 0 lens to begin (creates no
correction) and should be held in front of the
examiner's left eye when examining the client's right
eye and kept 1" from the client's eye for optimum
visualization. (A, B, D) are incorrect procedures.

What type of anemia is associated


with folate deficiency?
A. Microcytic
B. Pernicious
C. Megaloblastic
D. Iron deficiency

wheezes

C. Megaloblastic Megaloblastic anemia is nutritional anemia; large


immature red blood cells with a decreased oxygen-carrying capacity can
occur as a result of impaired DNA synthesis. Folic acid is used in the
synthesis of DNA and helps convert B12 to coenzyme form. Folic acid is
needed for growth and development of red blood cells. Microcytic
anemia is anemia with abnormally small erythrocytes (red blood cells)
in Hb. This anemia is associated with vitamin B6 (pyridoxine)
deficiency. Pernicious anemia is caused by a deficiency of vitamin B12.
Iron-deficiency anemia results from loss of blood or deficient intake of
iron foods or disease states in which the body does not absorb or utilize
iron as it should.

a form of rhonchus characterized by


continuous high-pitched squeaking
sound caused by rapid vibration of
bronchial walls.

When administering oxygen to a patient with COPD with


the potential for carbon dioxide narcosis, the nurse
should A. never administer oxygen at a rate of more than
2 L/min.
B. monitor the patient's use of oxygen to detect oxygen
dependency.
C. monitor the patient for symptoms of oxygen toxicity,
such as paresthesias.
D. use ABGs as a guide to determine what FIO2 level
meets the patient's needs.

D. use ABGs as a guide to determine


what FIO2 level meets the patient's
needs. It is critical to start oxygen at
low flow rates and then use ABGs as
a guide to determine what FIO2 level
is sufficient and can be tolerated.

When admitting a 45-year-old female with a


diagnosis of pulmonary embolism, the nurse will
assess the patient for which of the following risk
factors? (Select all that apply.)
A. Obesity
B. Pneumonia
C. Hypertension
D. Cigarette smoking

A,C,D Research has demonstrated an


increased risk of pulmonary embolism in
women associated with obesity, heavy
cigarette smoking, and hypertension. Other
risk factors include immobilization, surgery
within the last 3 months, stroke, history of
DVT, and malignancy.

When admitting a patient with the diagnosis of


asthma exacerbation, the nurse will assess for which
of the following potential triggers? (Select all that
apply.)
A. Exercise
B. Allergies
C. Emotional stress
D. Decreased humidity

A,B,C Although the exact mechanism of


asthma is unknown, there are several
triggers that may precipitate an attack.
These include allergens, exercise, air
pollutants, respiratory infections, drug
and food additives, psychologic factors,
and GERD.

When assessing a patient's nutritional-metabolic


pattern related to hematologic health, the nurse
would:
A. Inspect the skin for petechiae.
B. Ask the patient about joint pain.
C. Assess for vitamin C deficiency.
D. Determine if the patient can perform ADLs.

A. Inspect the skin for petechiae. Any changes in the


skin's texture or color should be explored when
assessing the patient's nutritional-metabolic pattern
related to hematologic health. The presences of
petechiae or ecchymotic areas could be indicative of
hematologic deficiencies related to poor nutritional
intake or related causes.

When assessing a patient's respiratory status, which


of the following nonrespiratory data are most
important for the nurse to obtain?
A. Height and weight
B. Neck circumference
C. Occupation and hobbies
D. Usual daily fluid intake

C. Occupation and hobbiesMany respiratory problems


occur as a result of chronic exposure to inhalation
irritants. Common occupational sources of inhalation
irritants include mines, granaries, farms, lawn care
companies, paint, plastics and rubber manufacture, and
building remodeling. Hobbies associated with inhalation
irritants include woodworking, metal finishing, furniture
refinishing, painting, and ceramics. Daily fluids, height,
and weight are more related to respiratory problems
secondary to cardiac issues.

When assessing a patient's sleep-rest pattern related


to respiratory health, the nurse would ask if the
patient: (Select all that apply.)
A. Has trouble falling asleep
B. Awakens abruptly during the night
C. Sleeps more than 8 hours per night
D. Has to sleep with the head elevated

A,B,D The patient with sleep apnea may have


insomnia and/or abrupt awakenings. Patients with
cardiovascular disease (e.g., heart failure that may
affect respiratory health) may need to sleep with the
head elevated on several pillows (orthopnea).
Sleeping more than 8 hours per night is not
indicative of impaired respiratory health.

When assessing lab values on a patient admitted with


septicemia, the nurse would expect to find:
A. Increased platelets
B. Decreased red blood cells
C. Decreased erythrocyte sedimentation rate (ESR)
D. Increased bands in the WBC differential (shift to
the left)

D. Increased bands in the WBC differential (shift to the


left) When infections are severe, such as in septicemia,
more granulocytes are released from the bone marrow as
a compensatory mechanism. To meet the increased
demand, many young, immature polymorphonuclear
neutrophils (bands) are released into circulation. WBCs
are usually reported in order of maturity, with the less
mature forms on the left side of a written report. Hence,
the term "shift to the left" is used to denote an increase in
the number of bands.

When assigning clients on a medical-surgical floor to a RN and a


LPN, it is best for the charge nurse to assign which client to the
LPN?
A. A child with bacterial meningitis with recent seizures.
B. An older adult client with pneumonia and viral meningitis.
C. A female client in isolation wiht meningococcal meningitis.
D. A male client 1 day post-op after drainage of a brain abscess.

B. Is the most stable. A, C, D have an


increased risk for elevated ICP.

When caring for a patient who is 3 hours


postoperative laryngectomy, the nurse's
highest priority assessment would be:
A. Airway patency
B. Patient comfort
C. Incisional drainage
D. Blood pressure and heart rate

A. Airway patency Remember ABCs


with prioritization. Airway patency is
always the highest priority and is
essential for a patient undergoing
surgery surrounding the upper
respiratory system.

When caring for a patient with COPD, the nurse identifies a nursing
diagnosis of imbalanced nutrition less than body requirements after
noting a weight loss of 30 lb. Which of the following would be an
appropriate intervention to add to the plan of care for this patient?
A. Teach the patient to use frozen meals at home that can be
microwaved.
B. Provide a high-calorie, high-carbohydrate, nonirritating, frequent
feeding diet.
C. Order fruits and fruit juices to be offered between meals.
D. Order a high-calorie, high-protein diet with six small meals a day.

D. Order a high-calorie, high-protein diet with six small meals a


day.Because the patient with COPD needs to use greater energy to
breathe, there is often decreased oral intake because of dyspnea. A
full stomach also impairs the ability of the diaphragm to descend
during inspiration, interfering with the work of breathing. Finally,
the metabolism of a high carbohydrate diet yields large amounts of
CO2, which may lead to acidosis in patients with pulmonary
disease. For these reasons, the patient with emphysema should
take in a high-calorie, high-protein diet, eating six small meals per
day.

When caring for a patient with metastatic cancer, the


nurse notes a hemoglobin level of 8.7 g/dl and
hematocrit of 26%. The nurse would place highest
priority on initiating interventions that will reduce
which of the following?
A. Fatigue
B. Thirst
C. Headache
D. Abdominal pain

A. Fatigue The patient with a low hemoglobin and


hematocrit (normal values approximately 13.5% to
17% and 40% to 54%, respectively) is anemic and
would be most likely to experience fatigue. This
symptom develops because of the lowered oxygencarrying capacity that leads to reduced tissue
oxygenation to carry out cellular functions.

When initially teaching a patient the supraglottic


swallow following a radical neck dissection, with
which of the following foods should the nurse begin?
A. Cola
B. Applesauce
C. French fries
D. White grape juice

A. ColaWhen learning the supraglottic swallow, it


may be helpful to start with carbonated beverages
because the effervescence provides clues about the
liquid's position. Thin, watery fluids should be
avoided because they are difficult to swallow and
increase the risk of aspiration. Nonpourable pureed
foods, such as applesauce, would decrease the risk of
aspiration, but carbonated beverages are the better
choice to start with.

When planning appropriate nursing interventions for a


patient with metastatic lung cancer and a 60-pack-year
history of cigarette smoking, the nurse recognizes that the
smoking has most likely decreased the patient's
underlying respiratory defenses because of impairment of
which of the following?
A. Reflex bronchoconstriction
B. Ability to filter particles from the air
C. Cough reflex
D. Mucociliary clearance

D. Mucociliary clearance Smoking


decreases the ciliary action in the
tracheobronchial tree, resulting in
impaired clearance of respiratory
secretions, chronic cough, and
frequent respiratory infections.

When planning patient teaching about emphysema, the nurse


understands that the symptoms of emphysema are caused by which
of the following?
A. Hypertrophy and hyperplasia of goblet cells in the bronchi
B. Collapse and hypoventilation of the terminal respiratory unit
C. An overproduction of the antiprotease alpha1-antitrypsin
D. Hyperinflation of alveoli and destruction of alveolar walls

D. Hyperinflation of alveoli and


destruction of alveolar walls In
emphysema, there are structural changes
that include hyperinflation of alveoli,
destruction of alveolar walls, destruction
of alveolar capillary walls, narrowing of
small airways, and loss of lung elasticity.

When preparing to administer an ordered blood


transfusion, the nurse selects which of the following
intravenous solutions to use when priming the blood
tubing?
A. 5% dextrose in water
B. Lactated Ringer's
C. 0.9% sodium chloride
D. 0.45% sodium chloride

C. 0.9% sodium chloride The blood set


should be primed before the transfusion with
0.9% sodium chloride, also known as normal
saline. It is also used to flush the blood
tubing after the infusion is complete to
ensure the patient receives blood that is left
in the tubing when the bag is empty.

When reviewing the results of a 83-year-old patient's


blood tests, which of the following findings would be
of most concern to the nurse?
A. Platelets of 150,000/l
B. Serum iron of 50 mcg/dl
C. Partial thromboplastin time (PTT) of 60 seconds
D. Erythrocyte sedimentation rate (ESR) of 35 mm in
1 hour

C. Partial thromboplastin time (PTT) of 60 seconds


In aging, the partial thromboplastin time (PTT) is
normally decreased, so an abnormally high PTT of 60
seconds is an indication that bleeding could readily
occur. Platelets are unaffected by aging, and 150,000
is a normal count. Serum iron levels are decreased
and the erythrocyte sedimentation rate (ESR) is
significantly increased with aging, as are reflected in
these values.

Which abnormal lab finding indicates


that a client with diabetes needs further
evaluation for diabetic nephropathy?
A. Hypokalemia
B. Microalbuminauria
C. Elevated serum lipids
D. Ketonuria

B. Microalbuminuria is the earliest sign


of nephropathy and indicates the need for
follow-up evaluation. Hyperkalemia (A)
is associated with end stage renal disease
caused by diabetic nephropathy. (C) may
be elevated in end stage renal disease.
(D) may signal the onset of DKA.

Which condition should the nurse anticipate as a


potential problem in a female client with a
neurogenic bladder?
A. Stress incontinence.
B. Infection.
C. Painless, gross hematuria.
D. Peritonitis.

B. Infection is the major complication resulting from


stasis of urine and subsequent catheterization. (A) is
the involuntary loss of urine through an intact
urethra as a result of suddenly increased pressure.
(C) is the most common symptom of bladder cancer.
(D) is the most common and serious complication of
peritoneal dialysis.

Which content about self-care should the nurse include in the


teaching plan of a client who has genital herpes? (Select all that
apply.)
A. Encourage annual physical and Pap smear.
B. Take antiviral medication as prescribed.
C. Use condoms to avoid transmission to others.
D. Warm sitz baths may relieve itching.
E. Use Nystatin suppositories to control itching.
F. Douche with weak vinegar solutions to decrease itching.

Which description of symptoms is characteristic of a


client with diagnosed with trigeminal neuralgia (tic
douloureux)?
A. Tinnitus, vertigo, and hearing difficulties.
B. Sudden, stabbing, severe pain over the lip and chin.
C. Unilateral facial weakness and paralysis.
D. Difficulty in talking, chewing, and swallowing.

A,B,C,D. (E) is specific for Candida


infections and (F) is used to treat
Trichomonas.

B. Trigeminal neuralgia is characterized by


paroxysms of pain, similar to an electric shock, in the
area innervated by one or more branches of the
trigeminal nerve.
A. Characteristic of Meniere's
C. Characteristic of Bell palsey
D. Characteristic of disorders of the hypoglossal (12th
cranial nerve)

Which of the following clinical manifestations would


the nurse expect to find during assessment of a
patient admitted with pneumococcal pneumonia? A.
Hyperresonance on percussion
B. Fine crackles in all lobes on auscultation
C. Increased vocal fremitus on palpation D. Vesicular
breath sounds in all lobes

C. Increased vocal fremitus on palpation.


A typical physical examination finding
for a patient with pneumonia is increased
vocal fremitus on palpation. Other signs
of pulmonary consolidation include
dullness to percussion, bronchial breath
sounds, and crackles in the affected area.

Which of the following conditions is


manifested by unexplained shortness of
breath and a high mortality rate?
A. Bleeding ulcer
B. Transient ischemia
C. Pulmonary embolism
D. MI

C. Pulmonary embolism A high mortality rate is


associated with a pulmonary embolism. A pulmonary
embolism is an obstruction of the pulmonary artery
caused by an embolus. It presents with hypoxia,
anxiety, restlessness, and shortness of breath.
Bleeding ulcers, MI, and transient ischemia are not
associated with such a high mortality rate.

Which of the following conditions or factors in a 64year-old patient diagnosed with head and neck cancer
most likely contributed to this health problem?
A. Patient's hobby is oil painting.
B. Patient's father also had head and neck cancer.
C. Patient uses chewing tobacco and drinks beer
daily.
D. Patient quit school at age 16 and has worked in a
butcher shop for more than 40 years.

C. Patient uses chewing tobacco and drinks beer daily.


Many environmental risk factors contribute to the
development of head and neck cancer, although the
actual cause is unknown. There does not appear to be a
genetic predisposition to this type of cancer. The two most
important risk factors are tobacco and alcohol use,
especially in combination. Other risk factors include
chewing tobacco, pipe smoking, marijuana use, voice
abuse, chronic laryngitis, exposure to industrial chemicals
or hardwood dust, and poor oral hygiene.

Which of the following foods is high


in iron?
A. Citrus fruits
B. Milk products
C. Yellow vegetables
D. Green leafy vegetables

D. Green leafy vegetables Green leafy


vegetables are high in iron. Foods cooked in
iron pots and foods such as liver (the richest
source), oysters, lean meats, kidney beans,
whole wheat bread, kale, spinach, egg yolk,
turnip tops, beet greens, carrots, apricots,
and raisins are also high in iron.

Which of the following instructions are most appropriate in the


home management of a patient who has undergone surgery for oral
cancer?
A. "You should drink plenty of fluids and eat foods you enjoy."
B. "It is normal to have some leakage of saliva from the suture
line."
C. "Lying in a prone position helps decrease swelling at the suture
line."
D. "You should avoid foods high in protein while your suture line is
healing."

A. "You should drink plenty of fluids and eat foods


you enjoy." For patients who have undergone
treatment for head and neck cancers, maintaining
adequate nutrition is a challenge. The nurse
encourages the patient to increase fluids to prevent
dehydration and liquefy secretions. These patients
are more likely to eat foods that they enjoy and can
tolerate.

Which of the following is a factor


significant in the development of anemia
in men?
A. Condom use
B. Large hemorrhoids
C. A diet high in cholesterol
D. Smoking one pack of cigarettes daily

B. Large hemorrhoids Gastrointestinal


(GI) tract bleeding is a common etiologic
factor in men and may result from peptic
ulcers, hiatal hernia, gastritis, cancer,
hemorrhoids, diverticula, ulcerative
colitis, or salicylate poisoning.

Which of the following nursing interventions is most


appropriate to enhance oxygenation in a patient with
unilateral malignant lung disease?
A. Positioning patient on right side.
B. Maintaining adequate fluid intake
C. Performing postural drainage every 4 hours
D. Positioning patient with "good lung down"

D. Positioning patient with "good lung down"


Therapeutic positioning identifies the best position for the
patient assuring stable oxygenation status. Research
indicates that positioning the patient with the unaffected
lung (good lung) dependent best promotes oxygenation
in patients with unilateral lung disease. For bilateral lung
disease, the right lung down has best ventilation and
perfusion. Increasing fluid intake and performing
postural drainage will facilitate airway clearance, but
positioning is most appropriate to enhance oxygenation.

Which of the following nursing interventions is of the


highest priority in helping a patient expectorate thick
secretions related to pneumonia?
A. Humidify the oxygen as able
B. Increase fluid intake to 3L/day if tolerated.
C. Administer cough suppressant q4hr.
D. Teach patient to splint the affected area.

B. Increase fluid intake to 3L/day if tolerated. Although


several interventions may help the patient expectorate
mucus, the highest priority should be on increasing fluid
intake, which will liquefy the secretions so that the patient
can expectorate them more easily. Humidifying the
oxygen is also helpful, but is not the primary intervention.
Teaching the patient to splint the affected area may also
be helpful, but does not liquefy the secretions so that they
can be removed.

Which of the following physical assessment findings


in a patient with pneumonia best supports the
nursing diagnosis of ineffective airway clearance? A.
Oxygen saturation of 85%
B. Respiratory rate of 28
C. Presence of greenish sputum
D. Basilar crackles

D. Basilar crackles The presence of


adventitious breath sounds indicates that
there is accumulation of secretions in the
lower airways. This would be consistent
with a nursing diagnosis of ineffective
airway clearance because the patient is
retaining secretions.

Which of the following positions is most


appropriate for the nurse to place a patient
experiencing an asthma exacerbation?
A. Supine
B. Lithotomy
C. High-Fowler's
D. Reverse Trendelenburg

C. High-Fowler'sThe patient
experiencing an asthma attack
should be placed in high-Fowler's
position to allow for optimal chest
expansion and enlist the aid of
gravity during inspiration.

Which of the following statements made by a nurse would indicate


proper teaching principles regarding feeding and tracheostomies?
A. "Follow each spoon of food consumed with a drink of fluid."
B. "Thin your foods to a liquid consistency whenever possible."
C. "Tilt your chin forward toward the chest when swallowing your
food."
D. "Make sure your cuff is overinflated before eating if you have
swallowing problems."

C. "Tilt your chin forward toward the chest when


swallowing your food." A nurse should instruct a patient
to tilt the chin toward the chest, which will close the
glottis and allow food to enter the normal passageway.
Ideally, foods should be of a thick consistency to enable
effective swallowing and reduce the risk of aspiration.
Overinflation of the cuff causes swallowing difficulties.
Fluids should be consumed in small amounts after
swallowing to prevent the risk of aspiration.

Which of the following statements made by a patient with COPD


indicates a need for further education regarding the use of an
ipratropium inhaler?
A. "I should rinse my mouth following the two puffs to get rid of the
bad taste."
B. "I should wait at least 1 to 2 minutes between each puff of the
inhaler."
C. "If my breathing gets worse, I should keep taking extra puffs of
the inhaler until I can breathe more easily."
D. "Because this medication is not fast-acting, I cannot use it in an
emergency if my breathing gets worse.

C. "If my breathing gets worse, I should keep taking


extra puffs of the inhaler until I can breathe more
easily." The patient should not take extra puffs of the
inhaler at will to make breathing easier. Excessive
treatment could trigger paradoxical bronchospasm,
which would worsen the patient's respiratory status.

Which of the following test results identify


that a patient with an asthma attack is
responding to treatment?
A. A decreased exhaled nitric oxide
B. An increase in CO2 levels
C. A decrease in white blood cell count
D. An increase in serum bicarbonate levels

A. A decreased exhaled nitric oxide.


Nitric oxide levels are increased in the
breath of people with asthma. A decrease
in the exhaled nitric oxide concentration
suggests that the treatment may be
decreasing the lung inflammation
associated with asthma.

While ambulating a patient with metastatic lung cancer, the nurse


observes a drop in oxygen saturation from 93% to 86%. Which of the
following nursing interventions is most appropriate based upon these
findings?
A. Continue with ambulation as this is a normal response to activity.
B. Move the oximetry probe from the finger to the earlobe for more
accurate monitoring during activity.
C. Obtain a physician's order for supplemental oxygen to be used during
ambulation and other activity.
D. Obtain a physician's order for arterial blood gas determinations to
verify the oxygen saturation.

C. Obtain a physician's order for


supplemental oxygen to be used during
ambulation and other activity. An oxygen
saturation level that drops below 90%
with activity indicates that the patient is
not tolerating the exercise and needs to
have supplemental oxygen applied.

While caring for a patient with respiratory disease, a nurse


observes that the oxygen saturation drops from 94% to 85% when
the patient ambulates. The nurse should determine that
A. supplemental oxygen should be used when the patient exercises.
B. ABG determinations should be done to verify the oxygen
saturation reading.
C. this finding is a normal response to activity and that the patient
should continue to be monitored.
D. the oximetry probe should be moved from the finger to the
earlobe for an accurate oxygen saturation measurement during
activity.

A. supplemental oxygen should be used


when the patient exercises.An oxygen
saturation lower than 90% indicates
inadequate oxygenation. If the drop is
related to activity of some type,
supplemental oxygen is indicated.

While obtaining the admission assessment data,


which of the following characteristics would a nurse
expect a patient with anemia to report?
A. Palpitations
B. Blurred vision
C. Increased appetite
D. Feeling of warm flushing sensation

A. Palpitations Patients experiencing moderate anemia


(hemoglobin [Hb] 6 to 10 g/dL) may experience dyspnea
(shortness of breath), palpitations, diaphoresis (profound
perspiration) with exertion, and chronic fatigue. Blurred
vision is associated in patients experiencing profound
anemia states. Anorexia is common in patients with
severe anemia, as well. Patients with anemia often appear
pale and complain of feeling cold because of
compensatory vasoconstriction of the subcutaneous
capillaries.

While teaching a patient with asthma about the appropriate use of


a peak flow meter, the nurse instructs the patient to do which of the
following?
A. Use the flow meter each morning after taking medications to
evaluate their effectiveness.
B. Empty the lungs and then inhale quickly through the
mouthpiece to measure how fast air can be inhaled.
C. Keep a record of the peak flow meter numbers if symptoms of
asthma are getting worse.
D. Increase the doses of the long-term control medication if the
peak flow numbers decrease.

C. Keep a record of the peak flow meter numbers if


symptoms of asthma are getting worse. It is
important to keep track of peak flow readings daily
and when the patient's symptoms are getting worse.
The patient should have specific directions as to when
to call the physician based on personal peak flow
numbers. Peak flow is measured by exhaling into the
meters and should be assessed before and after
medications to evaluate their effectiveness.

Вам также может понравиться